Re: [obm-l] Outra da Escola Naval 2004 - Geometria

2004-07-19 Por tôpico Rafael Ando
João Vitor wrote:
Geometria:
Considere um triangulo ABC, cujos oas lados AB, BC, AC medem 10cm, 25cm,
10sqrt(2)cm, respectivamente. Seja CH a Altura relativa ao lado AB. Com
centro
no pto médio do lado BC, traça-se uma circunferência que é tangente a CH no
pto T.
O comprimento desta Circunferência é:
A) 25pi/2
B) 25pi/4
C) 21pi/8
D) 5pi/2
E) 5pi/4
Abraços


 

tem certeza que os números são estes?? 10, 10sqrt(2) e 25 não constitui 
um triangulo, afinal de contas, 25>10+10sqrt(2)

=
Instruções para entrar na lista, sair da lista e usar a lista em
http://www.mat.puc-rio.br/~nicolau/olimp/obm-l.html
=


Re: [obm-l] Re: [obm-l] Racionalização...

2004-07-19 Por tôpico Rafael Ando
João Vitor wrote:
Como faço pra racionalizar:
1/(1-sqrt3(x))
um sobre um menos raíz cúbica de X
Como faço para racionalizar...qual o fator devo multiplicar..
Abraço
=
Instruções para entrar na lista, sair da lista e usar a lista em
http://www.mat.puc-rio.br/~nicolau/olimp/obm-l.html
=
Esta mensagem foi verificada pelo E-mail Protegido Terra.
Scan engine: VirusScan / Atualizado em 19/07/2004 / Versão: 1.5.2
Proteja o seu e-mail Terra: http://www.emailprotegido.terra.com.br/
E-mail classificado pelo Identificador de Spam Inteligente Terra.
Para alterar a categoria classificada, visite
http://www.terra.com.br/centralunificada/emailprotegido/imail/imail.cgi?+_u=rafael_ando&_l=1090274210.120515.12261.turvo.terra.com.br
 

lembrando que (x-y)(x^2+xy+y^2) = x^3 - y^3, multiplique o numerador e 
denominador por 1+x^(1/3)+x^(2/3), logo:
1/(1-x^(1/3)) = (1+x^(1/3)+x^(2/3))/(1-x)

=
Instruções para entrar na lista, sair da lista e usar a lista em
http://www.mat.puc-rio.br/~nicolau/olimp/obm-l.html
=


[obm-l] Re: [obm-l] Posição do número primo

2004-07-18 Por tôpico Rafael
E você tem razão, Domingos. Aliás, segundo o Mathematica também, o
51260592-ésimo primo é 1008560563.



- Original Message -
From: "Domingos Jr." <[EMAIL PROTECTED]>
To: <[EMAIL PROTECTED]>
Sent: Sunday, July 18, 2004 11:40 PM
Subject: Re: [obm-l] Posição do número primo


De acordo com o Mathematica este número não é primo! No entanto, não
tive saco pra esperar o Mathematica fatorá-lo.

>
2345654356252312553244531245423554235346774546244656243763445642456567847565
4516232498573489567423632655441332241523234165643547653445656344655634456156
4345656443656745477568978765846897846756497656809678505668940566784956574981
6564173253162313456456634575689347568934756543547653641705564815656488967786
89779897870786549454651



=
Instruções para entrar na lista, sair da lista e usar a lista em
http://www.mat.puc-rio.br/~nicolau/olimp/obm-l.html
=


[obm-l] Re: [obm-l] [obm-l] Polinomio - retificação

2004-07-18 Por tôpico Rafael
x1 + x2 + x3 + (a + bi) + (a - bi) = 39/8
2a = 39/8 - 7/8 = 32/8 = 4
a = 2

x1 . x1/2 . x1/4 = 1/6
x1 = cbrt(4/3), x2 = 1/cbrt(6), x3 = 1/[2cbrt(6)]

x1 . x2 . x3 . x4 . x5 = 5/16
(1/6).(a^2 + b^2) = 5/16
b^2 = 15/8 - 32/8 = -17/8
b = +- i * sqrt(17/8)

x4 = 2 + sqrt(17/8), x5 = 2 - sqrt(17/8)

x2 x3 x4 x5 + x1 x3 x4 x5 + x1 x2 x4 x5 + x1 x2 x3 x5 + x1 x2 x3 x4 = m/16

Com coragem, você encontrará: m = 35cbrt(6)/2 + 32/3.


Obs. 1: sqrt(x) = raiz quadrada de x, cbrt(x) = raiz cúbica de x

Obs. 2: o enunciado diz que (a + bi) é raiz complexa da equação, o que
garante que (a - bi) também o será, visto que os coeficientes da equação são
reais. Entretanto, o enunciado não diz que a raiz (a + bi) é complexa e não
real, assim como também não diz que a e b devem ser reais.


[]s,

Sampaio




- Original Message -
From: "Daniel Regufe" <[EMAIL PROTECTED]>
To: <[EMAIL PROTECTED]>
Sent: Sunday, July 18, 2004 12:03 PM
Subject: [obm-l] [obm-l] Polinomio - retificação


To achando algo de errado nessa questão ... mas tentem fazer pra mim ...
Seja p(x) = 16x^5 - 78x^4 + ... + mx - 5 um polinomio de coeficientes reais
tal que a equação p(x) = 0 admite mais do que uma raiz real e ainda, a + bi
é uma raiz complexa desta equação com a e b diferentes de zero. Sabendo-se
que 1/a é a razão da progressão geométrica formada pelas raízes
reais de p(x) = 0 e que a soma destas raízes vale 7/8 enquanto que o
produto é 1/6, o valor de m é : ...

[]´s
Regufe

=
Instruções para entrar na lista, sair da lista e usar a lista em
http://www.mat.puc-rio.br/~nicolau/olimp/obm-l.html
=


Re: [obm-l] Postulados ...

2004-07-18 Por tôpico Rafael
- Quando duas retas *concorrentes* determinam um ângulo reto são chamadas
*perpendiculares*.

- Quando duas retas *reversas* determinam um ângulo reto, elas são ditas
*ortogonais*.

[]s,

Rafael S.



- Original Message -
From: "Daniel Regufe" <[EMAIL PROTECTED]>
To: <[EMAIL PROTECTED]>
Sent: Sunday, July 18, 2004 5:43 PM
Subject: [obm-l] Postulados ...


Queria a opinião da galera da lista sobre o seguinte detalhe  ...

Vcs acham q retas perpendiculares eh um caso de retas ortogonais ou retas
ortogonais devem ser necessariamente reversas???

[]`s
Regufe



=
Instruções para entrar na lista, sair da lista e usar a lista em
http://www.mat.puc-rio.br/~nicolau/olimp/obm-l.html
=


Re: [obm-l] Questão de função anex

2004-07-16 Por tôpico Rafael Ando
[EMAIL PROTECTED] wrote:
Probleminha de função
grato e abraços
Junior

Esta mensagem foi verificada pelo E-mail Protegido Terra 
.
Scan engine: VirusScan / Atualizado em 14/07/2004 / Versão: 1.5.2
Proteja o seu e-mail Terra: http://www.emailprotegido.terra.com.br/


E-mail classificado pelo Identificador de Spam Inteligente.
Para alterar a categoria classificada, visite 
http://www.terra.com.br/centralunificada/emailprotegido/imail/imail.cgi?+_u=rafael_ando&_l=1089954369.793534.20326.pamplona.terra.com.br 
 



Esta mensagem foi verificada pelo E-mail Protegido Terra.
Scan engine: VirusScan / Atualizado em 14/07/2004 / Versão: 1.5.2
Proteja o seu e-mail Terra: http://www.emailprotegido.terra.com.br/
E-mail classificado pelo Identificador de Spam Inteligente Terra.
Para alterar a categoria classificada, visite
http://www.terra.com.br/centralunificada/emailprotegido/imail/imail.cgi?+_u=rafael_ando&_l=1089954369.793534.20326.pamplona.terra.com.br
se y^2+2y+f(x) = 0 tem raíz dupla, delta = 0 e portanto f(x) = 1
Fazendo as contas com f(x) = 1:
3^((x-2)/2)*9^((2x+1)/2x) = 3^((2x+5)/x) usando log (base 3) nos dois 
lados,
(x-2)/2 + (2x+1)/x = (2x+5)/x
(x-2)/2 = 4/x
x^2 - 2x - 8 = 0
soma das raizes = 2 (pelas relações de girard)
alternativa c

=
Instruções para entrar na lista, sair da lista e usar a lista em
http://www.mat.puc-rio.br/~nicolau/olimp/obm-l.html
=


Re: [obm-l] RE: [obm-l] MAIS DÚVIDAS!

2004-07-14 Por tôpico Rafael Ando
Rogerio Ponce wrote:
Olá Jorge e colegas da lista!
1o. problema
Como a interseção da superfície esférica com o plano é uma 
circunferência, um dos pontos tem que ser, necessariamente o ponto 
externo ao plano. E cada grupo de 3 pontos do plano definirá , com o 
ponto externo, uma nova esfera. Como temos 19 pontos no plano, podemos 
definir ao todo
19! / (16! * 3!)  = 969 superfícies esféricas

2o. problema
Ela acha que tanto faz uma coisa ou outra.
Portanto o valor esperado da aposta é $20.
Como a chance dos $2 é 1/3 , e a do perfume é 2/3 , podemos escrever que
$20 = $2 * 1/3 + P * 2/3
Logo, P = $29
[]'s
Rogério.

From: jorgeluis
OK! Guilherme! Esse tiro de misericórdia foi certeiro e em tempo 
real! Grato!!

Num plano são dados 19 pontos entre os quais não se encontram 3 
alinhados e nem
4 situados numa mesma circunferência. Fora do plano é dado mais um 
ponto.
Quantas superfícies esféricas existem cada uma passando por 4 dos 
pontos dados?

NOTA: Tenho impressão que este já saiu na lista, mas como não 
encontrei.?


A Sra. Black acha que é uma questão aleatória aceitar $20 em dinheiro 
ou fazer
uma aposta, extraindo uma bola de uma urna com 15 bolas brancas e 45 
vermelhas,
onde ela receberá $2 se a bola extraída for branca ou um perfume se a 
bola for
vermelha. Qual valor ela atribui ao perfume?


Um abraço à todos!
_
MSN Messenger: converse com os seus amigos online.  
http://messenger.msn.com.br

=
Instruções para entrar na lista, sair da lista e usar a lista em
http://www.mat.puc-rio.br/~nicolau/olimp/obm-l.html
=
Esta mensagem foi verificada pelo E-mail Protegido Terra.
Scan engine: VirusScan / Atualizado em 14/07/2004 / Versão: 1.5.2
Proteja o seu e-mail Terra: http://www.emailprotegido.terra.com.br/
E-mail classificado pelo Identificador de Spam Inteligente Terra.
Para alterar a categoria classificada, visite
http://www.terra.com.br/centralunificada/emailprotegido/imail/imail.cgi?+_u=rafael_ando&_l=1089851581.25514.2761.larissa.terra.com.br 


na verdade, a chance dos $2 eh 1/4 e o perfume 3/4... pelo mesmo 
raciocínio o perfume vale $26.

=
Instruções para entrar na lista, sair da lista e usar a lista em
http://www.mat.puc-rio.br/~nicolau/olimp/obm-l.html
=


[no subject]

2004-07-14 Por tôpico Rafael Alves da Silva
O QUE É LIMITE DE UMA FUNÇÃO?
_
MSN Hotmail, o maior webmail do Brasil.  http://www.hotmail.com
=
Instruções para entrar na lista, sair da lista e usar a lista em
http://www.mat.puc-rio.br/~nicolau/olimp/obm-l.html
=


[no subject]

2004-07-14 Por tôpico Rafael Alves da Silva
O QUE É LIMITE DE UMA FUNÇÃO?
_
MSN Hotmail, o maior webmail do Brasil.  http://www.hotmail.com
=
Instruções para entrar na lista, sair da lista e usar a lista em
http://www.mat.puc-rio.br/~nicolau/olimp/obm-l.html
=


Re: [obm-l] O que é seno hiperbolico?

2004-07-13 Por tôpico Rafael Ando
allanper wrote:
Alguém poderia me informar o que é sen hiperbólico?
Atenciosamente,
Allan

Esta mensagem foi verificada pelo E-mail Protegido Terra 
.
Scan engine: VirusScan / Atualizado em 08/07/2004 / Versão: 1.5.2
Proteja o seu e-mail Terra: http://www.emailprotegido.terra.com.br/


E-mail classificado pelo Identificador de Spam Inteligente.
Para alterar a categoria classificada, visite 
http://www.terra.com.br/centralunificada/emailprotegido/imail/imail.cgi?+_u=rafael_ando&_l=1089747866.764250.30527.gravatal.terra.com.br 
 

é a função senh(x) = (e^x - e^(-x))/2
=
Instruções para entrar na lista, sair da lista e usar a lista em
http://www.mat.puc-rio.br/~nicolau/olimp/obm-l.html
=


Re: [obm-l] Lado do Quadrado

2004-07-09 Por tôpico Rafael
Daniel,

Essa solução por Geometria Plana é muito bonita. Mas nem dá tanto
trabalho assim a de Analítica...

x^2 + (y - L)^2 = 1 (i)
(x - L)^2 + y^2 = 25   (ii)
(x - L)^2 + (y - L)^2 = 16   (iii)

De (ii) - (iii):2yL = L^2 + 9 <==> y = (L^2 + 9)/2L

De (iii) - (i):2xL = L^2 - 15 <==> x = (L^2 -15)/2L

Em (iii):(x - L)^2 + (y - L)^2 = 16
x^2 - 2xL + L^2 + y^2 - 2yL + L^2 = 16
x^2 + y^2 = 10
(L^2 -15)^2 + (L^2 + 9)^2 = 40L^2
L^4 - 26L^2 + 153 = 0
L^2 = 17   ou   L^2 = 9

Devido ao sistema de coordenadas adotado, x e y devem ser positivos. A única
solução, portanto, é L = sqrt(17).


[]s,

Rafael




- Original Message -
From: "Daniel Regufe" <[EMAIL PROTECTED]>
To: <[EMAIL PROTECTED]>
Sent: Friday, July 09, 2004 8:18 AM
Subject: Re: [obm-l] Lado do Quadrado


Temos tres incognitas soh. x, y e L ... Porem confesso q essa por analitica
nao foi uma solução padrão... da muito trabalho. Tem uma por plana ... vou
tentar passar pra vc. (...)

[]`

Regufe


=
Instruções para entrar na lista, sair da lista e usar a lista em
http://www.mat.puc-rio.br/~nicolau/olimp/obm-l.html
=


Re: [obm-l] Lado do Quadrado

2004-07-09 Por tôpico Rafael



São três incógnitas: x, y e L. Resolvendo o 
sistema, obtém-se L = sqrt(17).
 

  - Original Message - 
  From: 
  [EMAIL PROTECTED] 
  
  To: [EMAIL PROTECTED] 
  Sent: Friday, July 09, 2004 2:38 AM
  Subject: Re: [obm-l] Lado do 
  Quadrado
  Tres equações com tres incognitas ? Nao seria: 
  Tres equações com quatro incognitas: x^2, y^2, (y-L)^2 e (x-L)^2 
  Explique melhor, por favor.   



[obm-l] Re: [obm-l] Triângulo - problema

2004-07-08 Por tôpico Rafael
Sejam x a medida das partes iguais, P a intersecção do segmento de medida 7
com a hipotenusa, Q a intersecção do segmento de medida 9 com a hipotenusa,
y e z as medidas dos catetos, temos:

Pela lei dos cossenos em ABQ e AQP: z^2 + 7^2 = 2*x^2 + 2*9^2
Por Pitágoras em ABC: y^2 + z^2 = (3x)^2
Por Stewart em AQC: x*9^2 + x*y^2 = 2x(7^2 + x^2)

Subtraindo a segunda relação da primeira, obtemos: y^2 = 7x^2 - 113.

Simplificando a última e substituindo:
81 + 7x^2 - 113 = 98 + 2x^2 ==> 5x^2 = 130 ==> x = sqrt(26).

Logo, a hipotenusa mede 3sqrt(26).


[]s,

Rafael



- Original Message -
From: "Maurizio" <[EMAIL PROTECTED]>
To: <[EMAIL PROTECTED]>
Sent: Thursday, July 08, 2004 7:43 AM
Subject: [obm-l] Triângulo - problema


Tem-se um triângulo ABC retãngulo em A. A partir de A traçam-se dois
segmentos de reta que dividem a hipotenusa em três partes iguais e que
medem 7 e 9. Qual o valor da hipotenusa?

=
Instruções para entrar na lista, sair da lista e usar a lista em
http://www.mat.puc-rio.br/~nicolau/olimp/obm-l.html
=


Re: [obm-l] Duvida! :)

2004-07-07 Por tôpico Rafael Ando
Fabio Contreiras wrote:
 
Amigos, qual uma boa saida para esse problema? Desde ja obrigado!
 
  Todo numero real positivo pode ser escrito na forma 10^x . Tendo 
em vista que 8 = 10^0,90 , então o expoente x, tal que 125 = 10^x , 
vale aproximadamente?
 
a) 1,90 b) 2,10 c) 2,30 d) 2,50


Esta mensagem foi verificada pelo E-mail Protegido Terra 
.
Scan engine: VirusScan / Atualizado em 05/07/2004 / Versão: 1.5.2
Proteja o seu e-mail Terra: http://www.emailprotegido.terra.com.br/


E-mail classificado pelo Identificador de Spam Inteligente.
Para alterar a categoria classificada, visite 
http://www.terra.com.br/centralunificada/emailprotegido/imail/imail.cgi?+_u=rafael_ando&_l=1089168033.885644.3647.conventos.terra.com.br 
 

Se o problema foi apresentado em uma situação em que o aluno ainda não 
conhece logaritmo (pelo enunciado, parece ser verdade), então a solução 
sem logaritmo fica:

8x125 = 1000 = 10^3
mas 8x125 = 10^0,90 * 10^x = 10^(x+0,90)
Logo, 10^(x+0,90) = 10^3 e x+0,90 = 3, logo x = 2,10 (alternativa b)
=
Instruções para entrar na lista, sair da lista e usar a lista em
http://www.mat.puc-rio.br/~nicolau/olimp/obm-l.html
=


[obm-l] Re: [obm-l] Re: [obm-l] Análise Combinatória

2004-07-05 Por tôpico Rafael
^12+2184*x^13*y^10+2184*x^1
3*y^11+4504*x^21*y^12+1048*x^25*y^9+944*x^23*y^8+1290*x^28*y^13+1304*x^11*y^
9+268*x^7*y^13+96*x^5*y^9+96*x^5*y^10+4*x^32*y^11+16*x^30*y^10+y^6+x^4*y^14+
25*x^4*y^13+327*x^8*y^14+24*x^28*y^9+4*x^36*y^15+64*x^31*y^11+16*x^26*y^8+44
5*x^8*y^11+445*x^8*y^12+112*x^29*y^10+128*x^27*y^9+112*x^25*y^8+24*x^34*y^13
+16*x^35*y^14+64*x^23*y^7+16*x^33*y^12+896*x^29*y^13+51*x^4*y^9+51*x^4*y^11+
47*x^4*y^12+51*x^4*y^10+170*x^6*y^12+148*x^6*y^13+4*x*y^11+170*x^6*y^10+170*
x^6*y^11+4*x*y^8+4*x*y^9+4*x*y^10+10*x^2*y^10+10*x^2*y^11+6*x^2*y^12+4*x*y^6
+4*x*y^7+4*x^3*y^13+10*x^2*y^7+10*x^2*y^8+10*x^2*y^9+24*x^3*y^8+24*x^3*y^9+2
4*x^3*y^10+24*x^3*y^11+20*x^3*y^12+1716*x^12*y^13+1712*x^12*y^14+284*x^7*y^1
0+284*x^7*y^11+284*x^7*y^12+952*x^10*y^13+952*x^10*y^12+1286*x^28*y^12+1648*
x^27*y^11+1616*x^26*y^10+580*x^30*y^13+872*x^29*y^12+344*x^31*y^13+194*x^32*
y^14+3649*x^16*y^12+3649*x^16*y^13+3649*x^16*y^14+1304*x^11*y^12+1304*x^11*y
^13+2680*x^14*y^12+2680*x^14*y^13+2680*x^14*y^14+2680*x^14*y^11+664*x^9*y^12
+664*x^9*y^13+664*x^9*y^10+664*x^9*y^11+4060*x^17*y^14+4060*x^17*y^15+4060*x
^17*y^12+4060*x^17*y^13+4611*x^20*y^14+4611*x^20*y^16+4611*x^20*y^15+3176*x^
15*y^14+4611*x^20*y^13+4370*x^18*y^12+4370*x^18*y^13+4370*x^18*y^14+4370*x^1
8*y^15+4504*x^21*y^14+4504*x^21*y^15+4504*x^21*y^16+3399*x^24*y^15+3399*x^24
*y^16+3399*x^24*y^17+96*x^5*y^11+96*x^5*y^12+3425*x^20*y^18+3413*x^16*y^16+2
161*x^16*y^17+2184*x^13*y^13+2612*x^14*y^7+2184*x^13*y^14+439*x^8*y^13+3649*
x^16*y^15+1868*x^13*y^6+4346*x^18*y^16+3746*x^18*y^17+4*x^10*y^17+64*x^5*y^1
3+16*x^5*y^14+6*x^8*y^16+98*x^8*y^15+28*x^7*y^15+156*x^7*y^14+904*x^10*y^14+
544*x^10*y^15+128*x^10*y^16+1276*x^11*y^6+824*x^10*y^5+56*x^6*y^14+4*x^6*y^1
5+268*x^9*y^15+572*x^9*y^14+4509*x^20*y^17+3972*x^17*y^16+2996*x^17*y^17+262
8*x^14*y^15+100*x^32*y^12+80*x^33*y^13+1984*x^14*y^16+1280*x^11*y^14+2056*x^
13*y^15+2744*x^15*y^16+3160*x^15*y^15+40*x^34*y^14+188*x^32*y^13+224*x^30*y^
11+2948*x^15*y^7+y^7+439*x^8*y^5+256*x^7*y^4+3621*x^16*y^8+1483*x^12*y^15+96
*x^33*y^14+256*x^31*y^12+664*x^9*y^6+976*x^11*y^15

Procurando o coeficiente de x^12*y^13 encontramos 1716.

Boas férias!


[]s,

Rafael



- Original Message -
From: claudio.buffara
To: obm-l
Sent: Monday, July 05, 2004 3:52 PM
Subject: [obm-l] Re: [obm-l] Análise Combinatória


Oi, Carlos:

Eh que o seu enunciado foi um pouco longo, o que pode ter feito com que a
maioria das pessoas desistisse de le-lo ateh o fim.

O baralho tem:
4 A: 4 pontos cada
4 K: 3 pontos cada
4 Q: 2 pontos cada
4 J: 1 ponto cada
36 numeros: 0 pontos cada.

Voce quer saber o numero de maos de 13 cartas cuja soma eh 12 pontos.

Isso eh igual ao numero de solucoes do sistema:
4*(a1+a2+a3+a4) + 3*(k1+k2+k3+k4) + 2*(q1+q2+q3+q4) + (j1+j2+j3+j4) = 12;

a1+a2+a3+a4+k1+k2+k3+k4+q1+q2+q3+q4+j1+j2+j3+j4+n1+n2+ ... +n36 = 13,

onde o universo das 52 variaveis eh igual a {0,1}.

Isso eh igual ao coeficiente de x^12*y^13 na expansao de:
(1+4x^4y+6x^8y^2+4x^12y^3)*
(1+4x^3y+6x^6y^2+4x^9y^3+x^12y^4)*
(1+4x^2y+6x^4y^2+4x^6y^3+x^8y^4)*
(1+4xy+6x^2y^2+4x^3y^3+x^4y^4)*(1+y+y^2+y^3+y^4+y^5+y^6+y^7+y^8+y^9+y^10)

Repare que x controla a soma dos pontos e y o numero de cartas.

Infelizmente, eu estou de ferias no Rio de Janeiro, sem acesso a qualquer
tipo de software matematico, de modo que nao vou conseguir dar a resposta
numerica que voce deseja (fazer na mao nem pensar!)

[]s,
Claudio.


=
Instruções para entrar na lista, sair da lista e usar a lista em
http://www.mat.puc-rio.br/~nicolau/olimp/obm-l.html
=


Re: [obm-l] Trigonometria

2004-06-30 Por tôpico Rafael
cos(3x) = 4cos^3(x) - 3cos(x)
sen(2x) = 2sen(x)cos(x)

4cos^3(x) - 3cos(x) - 2sen(x)cos(x) = 0
cos(x)(4cos^2(x) - 3 - 2sen(x)) = 0

cos(x) = 0 ==> x = Pi/2 + k*Pi (k inteiro)

4cos^2(x) - 3 - 2sen(x) = 0
4(1 - sen^2(x)) - 3 - 2sen(x) = 0
4sen^2(x) + 2sen(x) - 1 = 0

Discriminante = 4 - 4*4*(-1) = 20 = 4*5

sen(x) = [-1 +- sqrt(5)]/4

x = Pi/10 + 2*Pi*k   ou   x = 9*Pi/10 + 2*Pi*k
ou
x = -3*Pi/10 + 2*Pi*k   ou  x = 13*Pi/10 + 2*Pi*k


Agora, você deve estar se perguntando: "Como ele descobriu esses ângulos?".
Eu pensei em calcular sen(x/2) de x = Pi/5 -- ângulo razoavelmente
conhecido, para o qual o cosseno é metade da razão áurea. Sabemos que
sen(Pi/10) = sen(Pi - Pi/10) -- as primeiras respostas -- e cos(Pi/5) =
sen(Pi/2 - Pi/5) = sen(3*Pi/10) = (1 + sqrt(5))/4, donde sen(-3*Pi/10) =
sen(Pi + 3*Pi/10) = -(1 + sqrt(5))/4 -- últimas respostas.

Considerando as imagens de seno e cosseno no intervalo [-1;1], os ângulos
sempre serão reais.


Um abraço,

Rafael



- Original Message -
From: [EMAIL PROTECTED]
To: [EMAIL PROTECTED]
Sent: Thursday, July 01, 2004 1:43 AM
Subject: [obm-l] Trigonometria


A equação cos(3x)=sen(2x) caiu no antigo vestibular da poliachei
soluções do tipo x=pi/2 + kpi (k pertencente aos inteiros ) As outras
soluções que achei , são x=arcsen(2+2sqrt(5))/-8 ou x=arcsen(2-2sqrt(5))/-8.
O gabarito que me mostraram tem como soluções coisas mais bem
comportadasDevo ter errado em contas...alguém pode ajudar?
Em tempo...O conjunto verdade da  equação  senx=1, pode ser dado por V={x
pertencente aos reais/ x=90 graus + k.360 graus, k pertencente aos
inteiros}? Explicando melhor...posso dizer x pertencente aos reais quando me
referir a graus??  Vi respostas em apostilas de cursinhos
  Obrigado a quem puder ajudar.
Korshinoi...


=
Instruções para entrar na lista, sair da lista e usar a lista em
http://www.mat.puc-rio.br/~nicolau/olimp/obm-l.html
=


[obm-l] geometria projetiva...

2004-06-29 Por tôpico Rafael Mendes de Oliveira
Olah, eh q eu estou aprendendo geometria projetiva agora, e eu queria saber 
onde eu posso encontrar material com teoria, exemplos e exercicios para eu 
poder fazer.

_
MSN Hotmail, o maior webmail do Brasil.  http://www.hotmail.com
=
Instruções para entrar na lista, sair da lista e usar a lista em
http://www.mat.puc-rio.br/~nicolau/olimp/obm-l.html
=


Re: [obm-l] OBM - 1997

2004-06-27 Por tôpico Rafael



Essa equação já caiu por diversas vezes em 
vestibulares (Fuvest, por exemplo) e a resolução gráfica é 
feita, ainda com precisão, apenas usando lápis e uma régua graduada. 
Usar um programa de computador é útil para quando você não tem idéia da 
função trabalhada, mas estamos falando de uma polinomial e de uma 
exponencial, as quais, por hipótese, você deveria conhecer.
A solução do Marcio é muito boa. 
Talvez não se ensinem limites, derivadas e integrais no ensino médio 
de hoje, mas, felizmente, nem sempre foi assim...
 
 

  - Original Message - 
  From: 
  [EMAIL PROTECTED] 
  
  To: [EMAIL PROTECTED] 
  Sent: Sunday, June 27, 2004 7:05 PM
  Subject: Re: [obm-l] OBM - 1997
  Sem duvida, 
  O Winplot da a resposta de imediato, mas foge da realidade de um 
  concurso, vestibular, olimpiada, etc... em que temos apenas um lapis, uma 
  caneta, borracha, etc... Em uma mensagem de 27/6/2004 00:57:33 
  Hora padrão leste da Am. Sul, [EMAIL PROTECTED] escreveu: 
  
  O jeito mais fácil de se resolver essa equação é 
graficamente. As soluções são x = - 0,7666... ou x = 2 ou x = 4. 

- Original Message - From: [EMAIL PROTECTED] To: [EMAIL PROTECTED] 
  Sent: Sunday, June 27, 2004 12:11 AM Subject: 
  [obm-l] OBM - 1997 Ola pessoal O numero de solucoes 
  reais da equacao x^2 = 2^x eh: a)0 b)1 c)2 d)3 e)4 



Re: [obm-l] OBM - 1997

2004-06-26 Por tôpico Rafael



O jeito mais fácil de se resolver essa equação é 
graficamente.
 
As soluções são x = - 0,7666... ou x = 2 ou x = 
4.
 
 

  - Original Message - 
  From: 
  [EMAIL PROTECTED] 
  
  To: [EMAIL PROTECTED] 
  Sent: Sunday, June 27, 2004 12:11 
AM
  Subject: [obm-l] OBM - 1997
  Ola pessoal 
  O numero de solucoes reais da equacao x^2 = 2^x eh: a)0 
  b)1 c)2 d)3 e)4 


Re: [obm-l] Ajuda em comb.

2004-06-26 Por tôpico Rafael Ando
[EMAIL PROTECTED] wrote:
Uma prova de atletismo é disputada por 9 atletas, dos quais apenas 4 
são brasileiros. Os resultados possíves para a prova, de modo que pelo 
menos um brasileiro fique numa das três primeiras colocações são em 
número de:

a)426 b)444 c)468 d)480 e) 504
Fiz da seguinte forma : primeiro os casos TOTAIS( sem tirar brasileiro 
nenhum) deu 504(9x8x7).
A seguir tirei todos os brasileiros e vi os casos possíveis, deram 
60(5x4x3). Depois, eu diminui e deu 444(letra B)

Só que um professor meu ao ver a questão ficou meio intrigado(não sei 
porque) e disse que ia dar uma olhada, iria refazer a questão porque 
achava que a resposta poderia ser letra A.

Alguem poderia ver se eu errei, me dizer onde está o erro e depois 
resolver a questão para mim caso esteja errado?
grato
Junior


Esta mensagem foi verificada pelo E-mail Protegido Terra 
.
Scan engine: VirusScan / Atualizado em 24/06/2004 / Versão: 1.5.2
Proteja o seu e-mail Terra: http://www.emailprotegido.terra.com.br/


E-mail classificado pelo Identificador de Spam Inteligente.
Para alterar a categoria classificada, visite 
http://www.terra.com.br/centralunificada/emailprotegido/imail/imail.cgi?+_u=rafael_ando&_l=1088210628.184797.1729.itajuba.terra.com.br 
 

sua resposta esta certa, dá 444.
=
Instruções para entrar na lista, sair da lista e usar a lista em
http://www.mat.puc-rio.br/~nicolau/olimp/obm-l.html
=


Re: [obm-l] calculo de área - acho que precisa de integral

2004-06-24 Por tôpico Rafael Ando
Bruno França dos Reis wrote:
-BEGIN PGP SIGNED MESSAGE-
Hash: SHA1
Ola
Como já disse, estou no 3o. ano, entao NAO aprendi cálculo integral 
"oficialmente"... Já li sobre em vários livros, entre eles Fundamentos de 
Matematica Elementar, do Iezzi.

Um problema que um amigo meu me havia proposto e que não sei se cheguei na 
resposta (ele tb nao sabe a resposta) era o seguinte:

Tem uma figura com uma área hachurada, devemos calcular o valor dessa área. 
Tentarei descrever a figura:
Um quadrado de lado "a". Com o compasso no vértice inferior esquerdo, traça-se 
um quarto de circunferência (C1) (interno ao quadrado), de raio "a" também. 
Com o compasso no centro do quadrado, traça-se uma circunferência (C2) de 
raio "a"/2.
Hachura-se a intersecção de C2 com a parte externa de C1.

qual a área em funcao de "a"?
fiz o seguinte:
um par de eixos sendo o eixo x coincidente com a diagonal do quadrado 
sup.esq.-inf.dir. O eixo y seria a outra diagonal do quadrado, de forma que 
há simetria. Calculei a equação das circunferência e fiz uma integral 
definida de -a/2 até a/2, subtraí o que sobrava do quadrado e da outra 
circunferência, e meu Maple disse algo que tenho medo de colar aqui. Era 
horrível! Tinha, se nao me engano, sqrt(-i)... coisas horriveis! muito, muito 
feio... nao sei como pode uma simples área ter dado uma resposta tao grande e 
feia.

é pra ser isso mesmo? errei em cálculos? tem outro jeito de fazer?
até
bruno
- -- 
Bruno França dos Reis
brunoreis at terra com br
icq: 12626000
gpg-key: http://planeta.terra.com.br/informatica/brunoreis/brunoreis.key

-BEGIN PGP SIGNATURE-
Version: GnuPG v1.2.4 (GNU/Linux)
iD8DBQFA2klSsHdDIT+qyroRAiXmAJ9q+4+YqgHxkdt6299ogaJ1AiKgFwCfWzws
t7bvExaTKKCdbXeRwxfgOWE=
=EfRJ
-END PGP SIGNATURE-
=
Instruções para entrar na lista, sair da lista e usar a lista em
http://www.mat.puc-rio.br/~nicolau/olimp/obm-l.html
=
Esta mensagem foi verificada pelo E-mail Protegido Terra.
Scan engine: VirusScan / Atualizado em 23/06/2004 / Versão: 1.5.2
Proteja o seu e-mail Terra: http://www.emailprotegido.terra.com.br/
E-mail classificado pelo Identificador de Spam Inteligente Terra.
Para alterar a categoria classificada, visite
http://www.terra.com.br/centralunificada/emailprotegido/imail/imail.cgi?+_u=rafael_ando&_l=1088049088.331401.17515.chui.terra.com.br
 

Definindo os eixos como vc definiu as equações das duas curvas serão y = 
sqrt(a^2/4-x^2) e y = sqrt(a^2-x^2) - a/sqrt(2). (considerando apenas a 
região acima do eixo, ie, onde y>0). A área entre 2 curvas é a integral 
da diferença entre as duas funções, ie, integral de sqrt(a^2/4-x^2) - 
sqrt(a^2-x^2) + a/sqrt(2). Integrando apenas o lado direito e depois 
multiplicando por 2 (pq a função é par, ie, f(-x) = f(x)), temos q os 
limites de integração serão 0 e a coordenada x do ponto onde as curvas 
se interceptam (que NÃO é a/2), e vale a*sqrt(2)/8 (para encontrar esse 
valor faça sqrt(a^2/4-x^2) = sqrt(a^2-x^2) - a/sqrt(2)) . Então a área é 
2* (integral de sqrt(a^2/4-x^2) - sqrt(a^2-x^2) + a/sqrt(2)), onde a 
integral é definida de 0 a a*sqrt(2)/8.
O valor da integral é a^2/64*(8arcsen(sqrt(2)/4) - 32arcsen(sqrt(2)/8) - 
8 + sqrt(31)-sqrt(7)) = 0,0356591*a^2 (aproximadamente), logo o valor da 
área é o dobro e vale portanto aproximadamente 0,071*a^2.
Se não errei em nenhum calculo deve ser isso tentei explicar o 
melhor possivel porque vc disse q não aprendera (oficialmente) calculo, 
mas não sei se deu pra entender...

Rafael
=
Instruções para entrar na lista, sair da lista e usar a lista em
http://www.mat.puc-rio.br/~nicolau/olimp/obm-l.html
=


Re: [obm-l] OUTRA QUESTÃO DE CONCURSO!

2004-06-24 Por tôpico Rafael Ando
[EMAIL PROTECTED] wrote:
OK! Rogério, Júnior e demais colegas! Gostaria da opinião dos colegas, pois não
consegui resolver a questão abaixo, talvez pelo fato do enunciado dúbio. Grato!
Uma distribuidora de discos está vendo se patrocina o disco de um novo grupo
vocal. Fará 50.000 discos e o distribuirá pelo país. O custo de fabricação e
distribuição de cada disco é de $2. Esses discos são vendidos ao preço unitário
de $4. A experiência com casos semelhantes mostra que em 30 por cento de tais
patrocínios vende-se 40 por cento dos discos, em 30 por cento deles 50 por
cento dos discos são vendidos, e em 40 por cento deles vende-se 60 por cento de
tais discos. Supondo que estas sejam todas as possibilidades possíveis
determine se a companhia deve empresar este disco.
Infelizmente, não tenho a resposta. Abraços!!!

__
WebMail UNIFOR - http://www.unifor.br.
=
Instruções para entrar na lista, sair da lista e usar a lista em
http://www.mat.puc-rio.br/~nicolau/olimp/obm-l.html
=
Esta mensagem foi verificada pelo E-mail Protegido Terra.
Scan engine: VirusScan / Atualizado em 24/06/2004 / Versão: 1.5.2
Proteja o seu e-mail Terra: http://www.emailprotegido.terra.com.br/
E-mail classificado pelo Identificador de Spam Inteligente Terra.
Para alterar a categoria classificada, visite
http://www.terra.com.br/centralunificada/emailprotegido/imail/imail.cgi?+_u=rafael_ando&_l=1088114707.436471.9707.itajuba.terra.com.br
 

A quantidade de discos que se espera serem vendidos são 
30%*40%+30%*50%+40%*60% = 51% do total, ou 25500 discos
Logo, a quantidade que se ganha com as vendas equivale a 25500*$4 = $102000
Como o custo é 5*$2 = $10 < $102000, vale a pena empresar o 
disco... bom pelo menos essa foi minha interpretação do enunciado.
=
Instruções para entrar na lista, sair da lista e usar a lista em
http://www.mat.puc-rio.br/~nicolau/olimp/obm-l.html
=


Re: [obm-l] corda+flecha=elipse??

2004-06-19 Por tôpico Rafael



Neylor,
 
Observe que é suficiente, para definir uma elipse, 
sabermos a localização dos focos e o comprimento do eixo maior, 
donde:
 
PF1 + PF2 = 2a   (definição)
 
Por outro lado, no seu problema, sabemos o 
comprimento do eixo maior (a medida da corda AB) e o comprimento do eixo menor 
(o dobro da medida da flecha). Satisfaremos à definição se, e somente 
se, F = sqrt((AB/2)^2 - f^2). Para isso (AB/2)^2 > f^2, ou ainda, AB > 2f, quando existirá 
elipse. Mas esta última condição sempre é verdadeira a menos que a corda AB 
seja diâmetro, o que resultaria numa circunferência.
 
 
[]s,
 
Rafael
 
 
 

  - Original Message - 
  From: 
  neylor farias magalhaes 
  To: [EMAIL PROTECTED] 
  Sent: Saturday, June 19, 2004 9:35 
  AM
  Subject: Re: [obm-l] 
  corda+flecha=elipse??
  
  neste trecho "  A elipse estará determinada se F 
  = sqrt((AB/2)^2 - f^2), sendo 'f' a medida da 
  flecha."   
  O lado direito da equacao sempre tera um resultado valido,eu acho que 
  para comparar com o foco eu teria que calculá-lo independentemente.
   
  O que eu procuro é a maneira de provar se o lg é uma elipse.
   
  Rafael <[EMAIL PROTECTED]> 
  wrote:
  



A elipse 
é o l.g. dos pontos de um plano cujas distâncias a dois pontos 
fixos (focos) desse plano têm soma constante. Sejam 'a' a medida do semi-eixo maior, 'b' a medida do 
semi-eixo menor, 'F' a distância dos focos ao centro da elipse, vale sempre: 

F^2 = a^2 - b^2.
 
No seu caso, o eixo maior é a corda AB, o 
semi-eixo menor tem o comprimento da flecha e o centro da elipse está no 
ponto médio de AB. A elipse estará 
determinada se F = sqrt((AB/2)^2 - f^2), 
    sendo 'f' a medida da flecha.
 
 
[]s,
 
Rafael
 
 

  - Original Message - 
  From: 
  neylor farias magalhaes 

  To: [EMAIL PROTECTED] 
  Sent: Friday, June 18, 2004 7:55 
  PM
  Subject: [obm-l] 
  corda+flecha=elipse??
  
  
  
  
  
  Desculpe por ter que anexar uma figura,mas ela é demostrativa.Uma 
  corda AB e uma flecha f ,de uma circunferencia ,defini uma 
  elipse?
   


Re: [obm-l] corda+flecha=elipse??

2004-06-18 Por tôpico Rafael



A elipse é o 
l.g. dos pontos de um plano cujas distâncias a dois pontos 
fixos (focos) desse plano têm soma constante. Sejam 'a' a medida do semi-eixo maior, 'b' a medida do semi-eixo menor, 
'F' a distância dos focos ao centro da elipse, vale sempre: 
F^2 = a^2 - b^2.
 
No seu caso, o eixo maior é a corda AB, o semi-eixo 
menor tem o comprimento da flecha e o centro da elipse está no ponto médio de 
AB. A elipse estará determinada se F 
= sqrt((AB/2)^2 - f^2), sendo 'f' a medida 
da flecha.
 
 
[]s,
 
Rafael
 
 

  - Original Message - 
  From: 
  neylor farias magalhaes 
  To: [EMAIL PROTECTED] 
  Sent: Friday, June 18, 2004 7:55 PM
  Subject: [obm-l] 
  corda+flecha=elipse??
  
  
  
  
  
  Desculpe por ter que anexar uma figura,mas ela é demostrativa.Uma 
  corda AB e uma flecha f ,de uma circunferencia ,defini uma 
  elipse?
   
  


Re: [obm-l] Eureka 01

2004-06-16 Por tôpico Rafael
Leia:

http://www.mat.puc-rio.br/~nicolau/olimp/obm-l.200203/msg00226.html


Um abraço,

Rafael



- Original Message -
From: [EMAIL PROTECTED]
To: [EMAIL PROTECTED]
Sent: Wednesday, June 16, 2004 12:20 AM
Subject: [obm-l] Eureka 01


Ola pessoal,


Os vertices de um decagono regular convexo ABC...J devem ser coloridos
usando-se apenas as cores verde, amarela e azul. De quantos modos isso
pode ser feito se vertices adjacentes nao podem receber a mesma cor?

a)1022
b)1024
c)1026
d)1524
e)1536

=
Instruções para entrar na lista, sair da lista e usar a lista em
http://www.mat.puc-rio.br/~nicolau/olimp/obm-l.html
=


Re: [obm-l] integral indefinida parte II

2004-06-01 Por tôpico Rafael
Não me atrevi a responder antes, pois imaginei que você quisesse saber
"como" integrar essa função, algo que eu não saberia explicar. É possível,
no entanto, conseguir o resultado por programas como o Maple, Mathematica,
Matlab etc.

A integral indefinida de f(x) = cos(x)*x^(3/2) é
{2*sqrt(x) [3*cos(x) + 2*x*sen(x)] - 3*sqrt(2*Pi)*C[sqrt(2x/Pi)]}/4,
em que C(z) é a integral C de Fresnel.

Há um bom resumo delas -- integrais de Fresnel -- aqui:
http://mathworld.wolfram.com/FresnelIntegrals.html


Um abraço,

Rafael de A. Sampaio




- Original Message -
From: levi queiroz
To: [EMAIL PROTECTED]
Sent: Tuesday, June 01, 2004 2:00 PM
Subject: [obm-l] integral indefinida parte II



Seja f(y)= ( y^( 3/2) ).cosy

Qual o valor da integral indefinida de f(y)?

recebi como resposta o seguinte: 0.6{seny. y^(3/2) -
1,5.seny.[2.y^(3/2)]/3}, no entanto se derivarmos a expressão acima não
obteremos f(y).

Ajuda, por favor!



=
Instruções para entrar na lista, sair da lista e usar a lista em
http://www.mat.puc-rio.br/~nicolau/olimp/obm-l.html
=


[obm-l] Unicidade de um trinômio

2004-05-25 Por tôpico Rafael
Pessoal,

Há alguns dias li este problema em outra lista:

"Provar que se um número irracional for zero de um trinômio do 2° grau,
x^2 + ax + b, com a e b racionais, então o trinômio será único."


..


Pelo teorema das raízes irracionais, sabemos que se m + sqrt(n) for raiz
irracional, então m - sqrt(n) também o será, com m e n racionais. Penso que
assim fique garantido que o trinômio existe, pois os seus coeficientes
serão racionais:

m + sqrt(n) + m - sqrt(n) = 2m
(m + sqrt(n))(m - sqrt(n)) = m^2 - n


No entanto, isso é suficiente para provar que ele é único? E, além disso,
desconsiderando a validade do teorema das raízes irracionais para o
problema, existiria uma outra forma de fazer a demonstração?


Já agradeço por qualquer sugestão, dica, comentário, ...


Obrigado,

Rafael de A. Sampaio


=
Instruções para entrar na lista, sair da lista e usar a lista em
http://www.mat.puc-rio.br/~nicolau/olimp/obm-l.html
=


[obm-l] Re: [obm-l] Combinatória

2004-05-15 Por tôpico Rafael
Não faz muito tempo que essa questão passou pela lista.

Veja a primeira das questões:
http://www.mail-archive.com/[EMAIL PROTECTED]/msg20517.html

E, antes de enviar um problema, não custa dar uma olhada nos arquivos da
lista...


Um abraço,

Rafael



- Original Message -
From: Pedro Costa
To: [EMAIL PROTECTED]
Sent: Saturday, May 15, 2004 1:58 PM
Subject: [obm-l] Combinatória


oi pessoal , ajude-me nesta questão:


De quantas maneiras 7 brinquedos podem ser divididos entre 3 crianças, se a
mais nova ganha 3 e cada uma das outras ganha 2?

=
Instruções para entrar na lista, sair da lista e usar a lista em
http://www.mat.puc-rio.br/~nicolau/olimp/obm-l.html
=


Re: [obm-l] Dominos e Fibonacci

2004-05-15 Por tôpico Rafael
Durante a semana, fiquei pensando sobre esse problema e alguns outros,
cheguei a algumas conclusões sobre as quais escrevo agora.

Suponhamos um tabuleiro 2x1:
 _
|_|
|_|

Há uma única possibilidade de disposição do dominó (suposto simétrico).

Agora, um tabuleiro 2x2:
 _ _
|_|_|
|_|_|

Temos duas possibilidades: dois dominós na horizontal ou dois dominós na
vertical.

Seja um tabuleiro 2x3:
_ _ _
|_|_|_|
|_|_|_|

Temos três possibilidades: três dominós na vertical, dois à esquerda na
horizontal e um na vertical, dois à direita na horizontal e um na vertical.

Para um tabuleiro 2x4:
_ _ _ _
|_|_|_|_|
|_|_|_|_|

Há cinco possibilidades: quatro dominós na vertical, dois à esquerda na
horizontal e dois na vertical, dois à direita na horizontal e dois na
vertical, dois dominós na horizontal no centro e um dominó na vertical em
cada extremo, quatro dominós na horizontal.

Curiosamente:

2x1 ---> 1 possibilidade ---> F(2) = 1
2x2 ---> 2 possibilidades ---> F(3) = 2
2x3 ---> 3 possibilidades ---> F(4) = 3
2x4 ---> 5 possibilidades ---> F(5) = 5

2xn ---> F(n+1) possibilidades,

em que F(n) é o enésimo número de Fibonacci.


Quanto ao problema que eu havia proposto ("probabilidade e quadradinhos"),
os tabuleiros são quadrados.

Seja um tabuleiro 2x2:
 _ _
|_|_|
|_|_|

Temos duas possibilidades na horizontal e outras duas na vertical, i.e.,
2*2 = 4 possibilidades.

Vamos ao 3x3:
 _ _ _
|_|_|_|
|_|_|_|
|_|_|_|

Temos 2*3 possibilidades na horizontal (duas possibilidades para cada
coluna). Na vertical, por simetria, temos outras 2*3 possibilidades, o que
nos dá 2*2*3 = 12 possibilidades.

Se tivéssemos um tabuleiro 4x4:
_ _ _ _
|_|_|_|_|
|_|_|_|_|
|_|_|_|_|
|_|_|_|_|

Teríamos 3*4 possibilidades na horizontal e, por simetria, 3*4
possibilidades na vertical: 2*3*4 = 24 possibilidades.

Analogamente, um tabuleiro nxn teria n(n-1) possibilidades na horizontal
(para cada coluna, n-1 possibilidades) e, por simetria novamente, n(n-1) na
vertical, o que nos traz o resultado que o Cláudio utilizou: 2n(n-1).


Na lista também foi proposto um problema sobre "sapos na escada" pelo
Anderson (também conhecido por Dirichlet --- agora com a identidade secreta
revelada). Reformulando o problema, em vez de uma escada, imaginemos tocas:

x |_|_|_|_|_|_|_|_|_|_|_|

O objetivo do sapo é, sem retroceder, partir de x e chegar novamente ao
chão. Supondo que haja apenas duas tocas, teremos:

- Do chão, o sapo salta e cai na 1a. toca, salta e cai na 2a., salta e vai
para o chão;
- Do chão, o sapo salta e cai na 1a. toca, salta e vai para o chão;
- Do chão, o sapo salta e cai na 2a. toca, salta e vai para o chão.

Para duas tocas, temos 3 possibilidades.

E se fossem três tocas?

- Do chão, o sapo salta e cai na 1a. toca, salta e cai na 2a., salta e cai
na 3a., salta e vai para o chão;
- Do chão, o sapo salta e cai na 1a. toca, salta e cai na 2a., salta e vai
para o chão;
- Do chão, o sapo salta e cai na 1a. toca, salta e cai na 3a., salta e vai
para o chão;
- Do chão, o sapo salta e cai na 2a. toca, salta e cai na 3a., salta e vai
para o chão;
- Do chão, o sapo salta e cai na 2a. toca, salta e vai para o chão.

Para três tocas, temos 5 possibilidades.

Mas... e se fossem quatro tocas? Em vez de enumerarmos as possibilidades,
observemos que o sapo, no máximo, dará 5 saltos. Se encontrarmos/contarmos
todas as partições de 5 em 1 e 2, partições estas em que a ordem é
importante, teremos:

5 = 1 + 1 + 1 + 1 + 1 =
= 2 + 1 + 1 + 1 =
= 1 + 2 + 1 + 1 =
= 1 + 1 + 2 + 1 =
= 1 + 1 + 1 + 2 =
= 2 + 2 + 1 =
= 2 + 1 + 2 =
= 1 + 2 + 2

Conseguimos oito partições, o que significa que o sapo terá oito
possibilidades quando houver quatro tocas.

Curiosamente de novo:

1 toca ---> 2 possibilidades ---> F(3)
2 tocas ---> 3 possibilidades ---> F(4)
3 tocas ---> 5 possibilidades ---> F(5)
4 tocas ---> 8 possibilidades ---> F(6),

em que F(n) é o enésimo número de Fibonacci.

Assim, quando houver n tocas (ou n degraus, no problema original), o sapo
terá F(n+2) maneiras de chegar ao chão (ou ao topo da escada).


Abraços,

Rafael de A. Sampaio




- Original Message -
From: "Claudio Buffara" <[EMAIL PROTECTED]>
To: "Lista OBM" <[EMAIL PROTECTED]>
Sent: Sunday, May 09, 2004 7:49 PM
Subject: [obm-l] Dominos e Fibonacci


De quantas maneiras podemos cobrir um tabuleiro 2xn com dominos?
Suponha que os dominos sao simetricos (ou seja, ambos os quadrados tem o
mesmo numero de bolinhas).


=
Instruções para entrar na lista, sair da lista e usar a lista em
http://www.mat.puc-rio.br/~nicolau/olimp/obm-l.html
=


Re: [obm-l] Probabilidade e quadradinhos

2004-05-09 Por tôpico Rafael
Cláudio,

Muito obrigado pela solução. Creio, assim, que a minha também esteja
correta. Tomando coragem para as contas:

P = [4*2 + 4(n-2)*3 + (n^2 - 4 - 4(n-2))*4]/[n^2(n^2 - 1)] =
= (8 + 12n - 24 + 4n^2 - 16 - 16n + 32)/[n^2(n^2 - 1)] =
= 4n(n - 1)/[n^2(n^2 - 1)] = 4/[n(n+1)]

...


A sua idéia de utilizar a escolha de um dominó foi ótima. Eu não estou
certo, mas esse resultado não tem a ver com os números de Fibonacci? É fácil
demonstrá-lo?


Um abraço e obrigado de novo,

Rafael de A. Sampaio




- Original Message -
From: "Claudio Buffara" <[EMAIL PROTECTED]>
To: <[EMAIL PROTECTED]>
Sent: Sunday, May 09, 2004 10:52 AM
Subject: Re: [obm-l] Probabilidade e quadradinhos


Eu acho que eh assim:

Numero de casos possiveis =
Numero de maneiras de se escolher 2 quadrados =
Binom(n^2,2) = n^2*(n+1)*(n-1)/2

Numero de casos favoraveis =
Numero de maneiras de se escolher dois quadrados com um lado em comum =
Numero de maneiras de se escolher um "domino" =
2*n*(n-1)

Probabilidade = 2*n*(n-1)/((n^2*(n+1)*(n-1)/2) = 4/(n*(n+1))


[]s,
Claudio.


=
Instruções para entrar na lista, sair da lista e usar a lista em
http://www.mat.puc-rio.br/~nicolau/olimp/obm-l.html
=


[obm-l] Re: [obm-l] Re: [obm-l] Re: [obm-l] Fatoração ( IMO )

2004-05-09 Por tôpico Rafael
Graças ao Fábio D. Moreira, agora sabemos que a "lorota" foi parcial...

Como o problema, pelo visto, interessou a várias pessoas da lista, eis a
demonstração que eu havia omitido:

(x + y)^7 - x^7 - y^7 =
= 7x^6y + 21x^5.y^2 + 35x^4.y^3 + 35x^3.y^4 + 21x^2.y^5 + 7x.y^6
= 7xy(x^5 + 3x^4.y + 5x^3.y^2 + 5x^2.y^2 + 3x.y^4 + y^5)
= 7xy[(x+y)(x^4-x^3.y+x^2.y^2-x.y^3+y^4+3xy(x^3+y^3)+5x^2.y^2(x+y)]
= 7xy[(x+y)(x^4-x^3.y+x^2.y^2-x.y^3+y^4+3xy(x+y)(x^2-xy+y^2)+5x^2.y^2(x+y)]
= 7xy(x+y)[x^4-x^3.y+x^2.y^2-x.y^3+y^4+3xy(x^2-xy+y^2)+5x^2.y^2]
= 7xy(x+y)[x^4 - x^3.y - x.y^3 + y^4 + 3xy(x^2 + xy + y^2)]
= 7xy(x+y)[x(x^3 - y^3) - y(x^3 - y^3) + 3xy(x^2 + xy + y^2)]
= 7xy(x+y)[(x-y)(x^3 - y^3) + 3xy(x^2 + xy + y^2)]
= 7xy(x+y)[(x-y)(x-y)(x^2 + xy + y^2) + 3xy(x^2 + xy + y^2)]
= 7xy(x+y)(x^2 + xy + y^2)(x^2 - 2xy + y^2 + 3xy)
= 7xy(x+y)(x^2 + xy + y^2)^2


Abraços,

Rafael de A. Sampaio





- Original Message -
From: "Fabio Contreiras" <[EMAIL PROTECTED]>
To: <[EMAIL PROTECTED]>
Sent: Sunday, May 09, 2004 4:00 PM
Subject: [obm-l] Re: [obm-l] Re: [obm-l] Fatoração ( IMO )


Valeu rafael, po então foi lorota do cara que me passou isso :) abraços!



- Original Message -
From: "Rafael" <[EMAIL PROTECTED]>
To: "OBM-L" <[EMAIL PROTECTED]>
Sent: Sunday, May 09, 2004 2:55 PM
Subject: [obm-l] Re: [obm-l] Fatoração ( IMO )


> Fábio,
>
> Acho pouco provável que esse tipo de exercício tenha caído numa IMO,
mas...
>
> (x + y)^7 - (x^7 + y^7) = 7xy(x + y)(x^2 + xy + y^2)^2
>
> Uma identidade semelhante foi usada por Lamé na demonstração do Último
> Teorema de Fermat para n = 7.
>
> (x + y + z)^7 - (x^7 + y^7 + z^7) =
> = 7(x+y)(x+z)(y+z)[(x^2 + y^2 + z^2 + xy + xz + yz)^2 + xyz(x + y + z)]
>
>
> Abraços,
>
> Rafael de A. Sampaio





=
Instruções para entrar na lista, sair da lista e usar a lista em
http://www.mat.puc-rio.br/~nicolau/olimp/obm-l.html
=


[obm-l] Re: [obm-l] Fatoração ( IMO )

2004-05-09 Por tôpico Rafael
Fábio,

Acho pouco provável que esse tipo de exercício tenha caído numa IMO, mas...

(x + y)^7 - (x^7 + y^7) = 7xy(x + y)(x^2 + xy + y^2)^2

Uma identidade semelhante foi usada por Lamé na demonstração do Último
Teorema de Fermat para n = 7.

(x + y + z)^7 - (x^7 + y^7 + z^7) =
= 7(x+y)(x+z)(y+z)[(x^2 + y^2 + z^2 + xy + xz + yz)^2 + xyz(x + y + z)]


Abraços,

Rafael de A. Sampaio





- Original Message -
From: Fabio Contreiras
To: [EMAIL PROTECTED]
Sent: Sunday, May 09, 2004 2:32 PM
Subject: [obm-l] Fatoração ( IMO )


Alguem tem ideia de como fatorar isso? Um Abraço!


( x + y )^7 - ( x^7 + y^7 )



=
Instruções para entrar na lista, sair da lista e usar a lista em
http://www.mat.puc-rio.br/~nicolau/olimp/obm-l.html
=


[obm-l] Probabilidade e quadradinhos

2004-05-09 Por tôpico Rafael
Pessoal,

Em outra lista, vi um problema interessante:

Uma folha quadrada de papel quadriculado contém n^2 quadradinhos (n >= 2).
Escolhendo-se, ao acaso, dois quadradinhos distintos, qual é a probabilidade
de que eles tenham um lado comum?

..


Há três "tipos" de quadradinhos: a) aqueles que estão nos cantos da folha e
possuem apenas dois outros quadradinhos adjacentes; b) aqueles que estão ao
longo das extremidades da folha e possuem apenas três quadradinhos vizinhos;
e c) aqueles que não estão em qualquer uma das posições anteriores da folha
e possuem quatro quadradinhos vizinhos.

Qualquer que seja n >= 2, teremos para:

a) 4 quadradinhos;
b) 4(n-2) quadradinhos;
c) n^2 - 4 - 4(n-2) quadradinhos.

A probabilidade de ocorrer (a) é:
p1 = 4/n^2 * 2/(n^2 - 1)

A probabilidade de ocorrer (b) é:
p2 = 4(n-2)/n^2 * 3/(n^2 - 1)

A probabilidade de ocorrer (c) é:
p3 = [n^2 - 4 - 4(n-2)]/n^2 * 4/(n^2 - 1)

E, se não errei até agora, a probabilidade pedida é:

P = p1 + p2 + p3


O que vocês acham?


Obrigado,

Rafael de A. Sampaio

=
Instruções para entrar na lista, sair da lista e usar a lista em
http://www.mat.puc-rio.br/~nicolau/olimp/obm-l.html
=


Re: [obm-l] Curiosidade OFF-TOPIC

2004-05-08 Por tôpico Rafael
Tenho a impressão de que se fez uma bagunça entre as unidades de tempo e as
unidades de medida de ângulo (ou de arco).

Um grau é igual a 1/360 de uma circunferência; um minuto, unidade de medida
de ângulo (ou de arco), é igual a 1/60 do grau; um segundo, unidade de
medida de ângulo (ou de arco), é igual a 1/60 do minuto. E, evidentemente,
hora é apenas uma unidade de tempo...


Abraços,

Rafael de A. Sampaio




- Original Message -
From: Fellipe Rossi
To: [EMAIL PROTECTED]
Sent: Friday, May 07, 2004 9:12 AM
Subject: Re: [obm-l] Curiosidade OFF-TOPIC


Sempre me perguntam isso no trabalho hehehe

Quanto a hora podemos pensar que um relógio de ponteiros tem 360 graus = 60
minutos de hora (ponteiro grande) e 30 graus = 1 hora (ponteiro menor)...
Então 1 grau não corresponde a 60 minutos de hora.

Eu não tenho certeza mas oq eu respondo é que Minutos significam 1/60 de
qualquer coisa. Como se fosse Décimo, ou Centésimo

Tipo uma banana pode ser dividida em 60 minutos de banana ehehe

Mas não tenho certeza se é isso.


- Original Message -
From: Alan Pellejero
To: [EMAIL PROTECTED]
Sent: Wednesday, May 05, 2004 11:49 AM
Subject: [obm-l] Curiosidade OFF-TOPIC


Sobre os ângulos:

1' = 60 "
60' = 1º

Essas são as relações, porém, há alguma relação entre o grau ser 60 minutos
e uma hora também o ser?

Muito obrigado,
Alan Pellejero




=
Instruções para entrar na lista, sair da lista e usar a lista em
http://www.mat.puc-rio.br/~nicolau/olimp/obm-l.html
=


[obm-l] Partições

2004-05-03 Por tôpico Rafael
Aos colegas da lista,

Estive estudando Partições em Mat. Discreta. O assunto é abordado, por
exemplo, nesta página:

http://mathworld.wolfram.com/Partition.html


É mencionada uma aplicação desse conceito para a resolução de equações
diofantinas. Alguém conhece outras?


Muito obrigado,

Rafael


P.S.: Enviei uma mensagem parecida faz algum tempo, não sei se houve
extravio...

=
Instruções para entrar na lista, sair da lista e usar a lista em
http://www.mat.puc-rio.br/~nicolau/olimp/obm-l.html
=


Re: [obm-l] Integral...

2004-04-27 Por tôpico Rafael



Uma outra sugestão é utilizar substituições da 
trigonometria hiperbólica. 
 
Você chegaria à expressão: sqrt(2) atanh[(tan(x/2) - 1)/sqrt(2)]
 
 
[]s,
 
Rafael
 
 

  - Original Message - 
  From: 
  Alan Pellejero 
  To: [EMAIL PROTECTED] 
  Sent: Tuesday, April 27, 2004 5:36 
  PM
  Subject: [obm-l] Integral...
  
  Olá amigos da lista, 
  pessoal, gostaria de saber se alguém tem uma "carta na manga para esse 
  aqui..."
   
  /|1/ (senx + cosx) dx 
  |
  /
   
  Eu fiz de uma maneira "corinthiana"...ou seja, deu 2 folhas!!!
  Queria saber se alguém tem uma solução são-paulina (inteligente, rápida, 
  objetiva, concisa...)
   
  Ps: esse não é o da hipociclóide de novo.rs
  valeu!
  té mais!
  Alan Pellejero
   


Re: [obm-l] Rafaeis

2004-04-25 Por tôpico Rafael
De fato, Morgado. Creio que a referência aos sobrenomes seja a forma mais
viável. Parece-me incoveniente guardar "logins" e repeti-los na lista,
devido à quantidade enorme de mensagens do tipo spam.




- Original Message -
From: Augusto Cesar de Oliveira Morgado
To: [EMAIL PROTECTED]
Sent: Sunday, April 25, 2004 6:16 PM
Subject: [obm-l] Rafaeis


Gente, nessa lista ha muitos Rafaeis, pelo menos 4. Se os chamamos de
Rafael, instala-se a confusão.
Fael
Rafael Sampaio ou Rafael cyberhelp
Rafael matdúvidas
Rafael San
O Rafael San podia nos informar o seu sobrenome (Santiago?).
Morgado

=
Instruções para entrar na lista, sair da lista e usar a lista em
http://www.mat.puc-rio.br/~nicolau/olimp/obm-l.html
=


[obm-l] Re: [obm-l] Progressão Aritmética..onde errei??

2004-04-25 Por tôpico Rafael



Daniel,
 
O seu erro está na contagem do número de termos que 
está somando. De 5 a (n+5), você tem (n+5)-5+1 termos, isto é, 
(n+1).
 
S(n+1) = (n+1)(8+4n+8)/2 = (n+1)(2n+8) = 2n^2 + 10n 
+ 8
 
Logo, A = 2 e B = 10.
 
 
[]s,
 
Rafael
 
 

  - Original Message - 
  From: 
  Daniel Silva 
  Braz 
  To: [EMAIL PROTECTED] 
  Sent: Sunday, April 25, 2004 11:32 
  PM
  Subject: [obm-l] Progressão 
  Aritmética..onde errei??
  
  Pessoal,
  Aqui vai um probleminha simples de PA que não consigo encontrar a mesma 
  resposta dada no livro..então alguém, por favor, poderia me dizer onde errei 
  ??
   
  
   
  (n + 5)
  Se S 4(x - 3) = An^2 + Bn + C, calcule 
  o valor de A + B
  (x 
  = 5)
  
  
  4(5 - 3) + 4(6 - 3) + 4(7 - 3) + ... + 4(n + 5 - 3) = An^2 + Bn + C
  8 + 12 + 16 + ... + 4(n + 2) = An^2 + Bn + C
   
  S(n + 5) = [(n + 5)(8 + 4n + 8)]/2 = An^2 + Bn + C
      
  [(n + 5)(4n + 16)]/2 = An^2 + Bn + C
      
  (4n^2 + 36n + 80)/2 = An^2 + Bn + C
      
  2n^2 + 18n + 40 = An^2 + Bn + C
   
      
  A + B = 20 
   
  No livro a resposta dada foi A + B = 12
   
  Daniel S. Braz


[obm-l] Re: [obm-l] Dúvida

2004-04-18 Por tôpico Rafael



Vale a pena ler:
 
http://www.astro.iag.usp.br/~mpallen/milenio.htm
 
 

  - Original Message - 
  From: 
  [EMAIL PROTECTED] 
  
  To: [EMAIL PROTECTED] 
  Sent: Sunday, April 18, 2004 11:31 
  PM
  Subject: Re: [obm-l] Dúvida
  2000/10 + 1 
  = 201 Estamos na decada 201 2000/100 + 1 = 21 Estamos no seculo 21 
  2000/1000 + 1 = 3 Estamos no terceiro milenio Acrescentei 1 em 
  cada equacao, pois passamos de 2000. Em uma mensagem de 18/4/2004 
  23:00:45 Hora padrão leste da Am. Sul, [EMAIL PROTECTED] 
  escreveu: 
  Em que década estamos atualmente ? Em que século 
estamos atualmente ? 



[obm-l] Re: [obm-l] Combimatória

2004-04-17 Por tôpico Rafael
Anselmo,

Sobre o primeiro problema, observe que a ordem na escolha dos brinquedos não
importa para o grupo formado, mas importa a ordem na entrega dos brinquedos.
Por exemplo: suponha que três dos sete brinquedos sejam uma boneca, um
carrinho e uma bola; se você escolher o carrinho, a bola e a boneca, ou a
boneca, a bola e o carrinho, você essencialmente terá o mesmo grupo, no
entanto, se você der esses brinquedos para um menino, talvez (?) ele fique
chateado por ter recebido uma boneca, mas uma menina que não goste de
carrinhos ou futebol ficaria bem mais chateada se recebesse os mesmos
brinquedos -- a ordem na entrega fará diferença. Ou seja, dos sete
brinquedos devemos escolher 3 para a C1, 2 para a C2, 2 para a C3, sendo Cn
a enésima criança, não necessariamente nessa ordem. Vamos esquematizar:

1º caso:

Fixar os três brinquedos de C1 como os primeiros a serem escolhidos e
permutar C2 e C3: 2!*C(7,3)*C(4,2)*C(2,2)

2° caso:

Fixar os dois brinquedos de C1 como os intermediários a serem escolhidos e
permutar C2 e C3 nas posições inicial e final: 2!*C(7,2)*C(5,3)*C(2,2)

3º caso:

Fixar os dois brinquedos de C1 como os últimos a serem escolhidos e permutar
C2 e C3:2!*C(7,2)*C(5,2)*C(3,3)

A resposta será a soma de todos os casos anteriores:

2! [C(7,3)C(4,2)C(2,2) + C(7,2)C(5,3)C(2,2) + C(7,2)C(5,2)C(3,3)] = 1260


Sobre o segundo problema, observe que não faz diferença a ordem em que o
aluno responde às questões: se ele responder 1ª., 2ª., 3ª. ou 3ª., 1ª. e 2ª.
serão as mesmas questões respondidas. O aluno citado deve responder a
quaisquer quatro das cinco primeiras questões *e* responder outras três
questões das cinco restantes para completar as oito que precisa responder.
Assim, C(5,4)*C(5,3) = 50. (Também não importa se você escolhe primeiro as
cinco últimas e depois as cinco primeiras, serão as mesmas questões
escolhidas).


Abraços,

Rafael de A. Sampaio





- Original Message -
From: "anselmo.ceara" <[EMAIL PROTECTED]>
To: "obm-l" <[EMAIL PROTECTED]>
Sent: Saturday, April 17, 2004 3:53 PM
Subject: [obm-l] Combimatória


Pesso ajudo no seguinte:

i) De quantas maneiras 7 brinquedos podem ser divididos
entre 3 crianças, se a mais nova ganha 3 e cada uma das
outras ganha 2?

ii) Um aluno precisa responder 8 das 10 questões de um
exame. Quantas alternativas ele tem , se ele deve
responder 4 das 5 primeiras questões?


:) Anselmo



=
Instruções para entrar na lista, sair da lista e usar a lista em
http://www.mat.puc-rio.br/~nicolau/olimp/obm-l.html
=


[obm-l] Re: [obm-l] dúvida

2004-04-17 Por tôpico Rafael
Leia:

http://www.mat.puc-rio.br/~nicolau/olimp/obm-l.200404/msg00299.html


Abraços,

Rafael



- Original Message -
From: TSD
To: [EMAIL PROTECTED]
Sent: Saturday, April 17, 2004 6:42 PM
Subject: [obm-l] dúvida


poderiam ajudar por favor!
simplificar :

1) "a" está elevado a tudo isto aí => a^ ([log(loga)]/loga)

2)  a ^ (loga^b.logb^c.logc^d) a base é oque está antes do ^

=
Instruções para entrar na lista, sair da lista e usar a lista em
http://www.mat.puc-rio.br/~nicolau/olimp/obm-l.html
=


[obm-l] Re: [obm-l] Re:_[obm-l]_Equação_Trigonométrica!

2004-04-17 Por tôpico Rafael
Sim, você tem razão. As contas é que são um pouquinho feias, mas...

Sabemos que sen(x) = (e^(ix) - e(-ix)) / (2i).

Seja y = e^(ix), para sen(x) = (3 + sqrt(7)*i)/4, temos:

y - 1/y = (-sqrt(7) + 3i)/2 ==> 2y^2 + (sqrt(7) - 3i)y - 2 = 0

D = (sqrt(7) - 3i)^2 - 4*2*(-2) = 14 - 6i*sqrt(7)

y = (-sqrt(7) + 3i +- (14 - 6i*sqrt(7))^(1/2)) / 4

Como (14 - 6i*sqrt(7))^(1/2) = sqrt(4sqrt(7) + 7) - i*sqrt(4sqrt(7) - 7),

x = -i*ln((sqrt(4sqrt(7) + 7) - sqrt(7) + (3 - sqrt(4sqrt(7) - 7))*i) / 4) =
= 0,621 + 0,743*i (aprox.)

ou

x = -i*ln((-sqrt(4sqrt(7) + 7) - sqrt(7) + (3 + sqrt(4sqrt(7) - 7)*i) / 4) =
= 2,521 - 0,743*i  (aprox.)


Analogamente, encontrar-se-iam os valores de x para sen(x) =
(3-sqrt(7)*i)/4, ressaltando, ainda, que não estamos resolvendo aquela
equação em C, mas encontrando os valores de x para os quais sen(x) é
complexo não-real.


Abraços,

Rafael



- Original Message -
From: Johann Peter Gustav Lejeune Dirichlet
To: [EMAIL PROTECTED]
Sent: Friday, April 16, 2004 1:36 PM
Subject: Re: [obm-l] Re:_[obm-l]_Equação_Trigonométrica!


Se voce definir seno em complexos fica facil.Acho que e^it=cos t+ i*sen t




=
Instruções para entrar na lista, sair da lista e usar a lista em
http://www.mat.puc-rio.br/~nicolau/olimp/obm-l.html
=


[obm-l] Re: [obm-l] Re: [obm-l] Farpas, Problemas e Formação

2004-04-17 Por tôpico Rafael
Cláudio,

Estranhamente, ou nem tanto, não foi bem a "curiosidade" a sua aparente
intenção quando escreveu: "(...) não vejo razão para se esconder a própria
formação, como o Rafael parece estar fazendo". Se você realmente estivesse
apenas interessado em conhecer as pessoas nesta lista, seria, de fato,
natural escrever uma mensagem com este tópico especificamente.
Não me cabe aqui discutir a sua simpatia ou antipatia por mim ou pelo tom
que você julga que eu tenha. Se responder às mensagens com um mínimo de
justificativa ou didática para os "cálculos" posteriores, e não simplesmente
mostrá-los, é ter um "tom professoral", talvez isso tenha ocorrido com
algumas das minhas mensagens.
E, satisfazendo à enquete por você lançada, não faria muito sentido que eu
fosse professor de Matemática e, assim mesmo, cometesse enganos banais
certas vezes. A Matemática está vinculada explicitamente à Computação, que é
meu objeto de estudo e pesquisa atualmente.

Por fim, de uma próxima vez, em vez de insinuações ofensivas, espero que se
manifeste a cordialidade, procurando-se, como mencionado, o propósito da
lista em problemas e situações matemáticas.


Ab imo corde,

Rafael de A. Sampaio





- Original Message -
From: "Claudio Buffara" <[EMAIL PROTECTED]>
To: <[EMAIL PROTECTED]>
Sent: Friday, April 16, 2004 1:46 PM
Subject: Re: [obm-l] Re: [obm-l] Farpas, Problemas e Formação


Oi, Rafael:

A sua formação não me é significativa de forma alguma e se você não quiser
falar a respeito, não tem problema nenhum. Não se toca mais nesse assunto.

Eu apenas estou curioso quanto à demografia da lista obm-l, ou seja, se os
participantes são alunos, professores, pesquisadores ou apenas amadores (que
é o meu caso). De fato, através da lista eu passei a conhecer várias pessas
muito legais e que, assim como eu, gostam de matemática. Entre elas, estão
alguns professores. Pelo tom um tanto quanto professoral das suas mensagens,
eu imaginei que você pudesse lecionar em alguma escola. Assim, nada mais
natural do que perguntar sobre sua formação. Mas, pra evitar qualquer
mal-entendido, retiro a pergunta.

[]s,
Claudio.

=
Instruções para entrar na lista, sair da lista e usar a lista em
http://www.mat.puc-rio.br/~nicolau/olimp/obm-l.html
=


Re: [obm-l] Problemas de Olimpiadas

2004-04-17 Por tôpico Rafael
Concordo com você, Paulo. E, certamente, a maioria estará disposta a
participar, visando ao propósito principal da lista.


Um abraço,

Rafael de A. Sampaio




- Original Message -
From: "Paulo Santa Rita" <[EMAIL PROTECTED]>
To: <[EMAIL PROTECTED]>
Sent: Saturday, April 17, 2004 12:47 PM
Subject: RE: [obm-l] Problemas de Olimpiadas


Ola Pessoal,

A ideia do Claudio e muito boa. Inclusive resgata o objetivo original desta
lista.  Dentro da
disponibilidade de tempo de cada um, todos devem participar.

Um Abraco
Paulo Santa Rita
7,1247,170404

=
Instruções para entrar na lista, sair da lista e usar a lista em
http://www.mat.puc-rio.br/~nicolau/olimp/obm-l.html
=


Re: [obm-l] Provas IME 1996/1997-2003/2004

2004-04-17 Por tôpico Rafael
É uma boa idéia, sim. Concordo em participar também.

Só é uma pena o Yahoo não armazenar os anexos das mensagens, que poderiam
vir a surgir na resolução dos problemas...



- Original Message -
From: [EMAIL PROTECTED]
To: [EMAIL PROTECTED]
Sent: Saturday, April 17, 2004 2:15 AM
Subject: Re: [obm-l] Provas IME 1996/1997-2003/2004


E se criassemos uma lista la no yahoo para resolucoes de provas de
vestibulares, o que acham ?
Os grupos do yahoo possuem uma area para postagem de ARQUIVOS (*gif, *doc,
*bmp, etc...) e, sendo assim, seria o substrato das provas resolvidas.
Conforme vao aparecendo nos sites as provas dos vestibulares do pais e sendo
publicado os gabaritos, o que nos resta e tentarmos soluciona-las e
publica-las no nosso grupo. Mas nao adianta criarmos o grupo la no yahoo e
ninguem participar...quero saber da opiniao de voces, caso concordem em
participar, eu ou qualquer um que se disponha criara a lista e quem quiser
se inscrevera.




=
Instruções para entrar na lista, sair da lista e usar a lista em
http://www.mat.puc-rio.br/~nicolau/olimp/obm-l.html
=


[obm-l] Combinatória - Partições

2004-04-16 Por tôpico Rafael
Pessoal,

Eu estava estudando Partições em Matemática Discreta. O assunto é abordado,
por exemplo, nesta página:

http://mathworld.wolfram.com/Partition.html


É mencionada uma aplicação desse conceito para a resolução de equações
diofantinas. Alguém conhece outras?


Muito obrigado,

Rafael de A. Sampaio



=
Instruções para entrar na lista, sair da lista e usar a lista em
http://www.mat.puc-rio.br/~nicolau/olimp/obm-l.html
=


[obm-l] Re: [obm-l] Equação Trigonométrica!

2004-04-16 Por tôpico Rafael
Carlos,

Se sen(x) > 0, então 2 sen^2(x) + 3 sen(x) - 2 = 0

D = 3^2 - 4*2*(-2) = 9 + 16 = 25
sen(x) = (-3 +- 5)/4 ==> sen(x) = -2   ou  sen(x) = 1/2

Como sen(x) > 0, então sen(x) = 1/2.

Logo, x = Pi/6 + 2*k*Pi   ou   x = 5Pi/6 + 2*k*Pi,
sendo k inteiro.

Se sen(x) < 0, então: - 2 sen^2 + 3 sen(x) - 2 = 0

D = 3^2 - 4*(-2)*(-2) = 9 - 16 = -7

Por D < 0, sabemos que as raízes dessa equação são valores para os quais
sen(x) é complexo não-real. Veja:

sen(x) = [-3 +- sqrt(7)*i] / (-4) ==>
==> sen(x) = [3 + sqrt(7)*i]/4   ou   sen(x) = [3 - sqrt(7)*i]/4


Encontrar os valores de x que satisfazem a essas equações imagino que não
seja fácil, mas o exercício pede que você resolva em R. Assim, o conjunto
solução é aquele mesmo que você mencionou.


Abraços,

Rafael



- Original Message -
From: Carlos Alberto
To: [EMAIL PROTECTED]
Sent: Friday, April 16, 2004 9:28 AM
Subject: [obm-l] Equação Trigonométrica!


Resolva em R, a seguinte equação.

2 . senx . |senx| + 3 . senx = 2

Desculpa a pertinência em enviar questão que foge do escopo da lista.
Mas não tenho muitos locais para recorrer.
Segue abaixo minha resolução que eu não considerei tanto correta.

Resolução.

|senx| < 0 ou
|senx| > 0

logo, para

|senx| < 0
-2 sen^2 x + 3 sen x - 2 = 0

Considerando sen x = t ( * )

-2 t^2 + 3t - 2 = 0

9 - 16 = - 7 ---> Não possui raízes reais, logo não convém.

|senx| > 0

2 sen^2 x + 3 sen x - 2 = 0
Considerando sen x = t

2 t^2 + 3 t - 2 = 0
t' = - 2 (**)
t" = 1/2 (***)

Substituindo (*) em (**) e (***) temos,

senx = 1/2
senx = sen pi/6

x = pi/6 + 2kpi ou
x = 5pi/6 + 2kpi

Bom... até aqui tudo bem!!!

A Solução do livro é:
V = { x pert R | x = pi/6 + 2kpi ou x =  5 pi/6 + 2kpi}

O que ocorre com o "sen x = -2"??

Reparei no livro que nºs > 1 e nºs < 1 são "aparentemente desconsiderados".

O pq disso? Eu imaginei sendo que a imagem de sen x = [-1, 1].

Mas não sei é realmente isso que ocorre.

Pois por outro lado eu enxergaria
sen x = -2  como,
sen x = -2 . sen pi/2

Alguém poderia esclarecer minha dúvida, e conferir se eu fiz algo errado na
resolução.

Desde já agradeço a atenção!!!

[ ], s Carlos


=
Instruções para entrar na lista, sair da lista e usar a lista em
http://www.mat.puc-rio.br/~nicolau/olimp/obm-l.html
=


[obm-l] Re: [obm-l] Farpas, Problemas e Formação

2004-04-16 Por tôpico Rafael
Cláudio,

Concordo com a sua mensagem até a parte que me diz respeito. Em nenhum
momento, pretendi esconder a minha formação, mas também não pensei que a
minha lhe era tão significativa, ou que a minha e a sua fossem assunto para
e-mails nesta lista. Além disso, se os mal-entendidos surgem, certamente não
são intencionais e são contornáveis, tanto que já estão desfeitos.


Um abraço,

Rafael de A. Sampaio




- Original Message -
From: claudio.buffara
To: obm-l
Sent: Friday, April 16, 2004 9:23 AM
Subject: [obm-l] Farpas, Problemas e Formação


Pessoal:

Vamos parar com essas trocas de farpas, as quais, além de desagradáveis e
off-topic, são contra-producentes. Muito melhor é concentrarmos a nossa
energia na resolução dos problemas ainda em aberto na lista, tais como, por
exemplo, o que o Danilo mandou há alguns dias:

Exiba uma função f:[0,+infinito) -> R, de classe C^1 (ou seja, com derivada
contínua) e tal que qundo t tende a +infinito, f''(t) + (f'(t))^2 tende
a -infinito.
(Danilo: por favor me corrija se o enunciado estiver errado)

Além desse, eu tenho um outro de combinatória em duas partes:
Parte 1: Prove que, se escolhermos quaisquer 90 elementos distintos do
conjunto {1,2,3,...,2004}, existirão quatro elementos distintos, dentre os
90 escolhidos, tais que a soma de dois deles é igual à soma dos outros dois.

Parte 2: Determine o menor N tal que, se escolhermos quaisquer N elementos
distintos do conjunto {1,2,3,...,2004}, existirão quatro elementos
distintos, dentre os N escolhidos, tais que a soma de dois deles é igual à
soma dos outros dois.

Em tese, existe também uma parte 3, que é igual à parte 2, mas com o
conjunto sendo {1,2,...,M}.

A parte 1 eu consegui fazer, mas gostaria de ver soluções diferentes da
minha, pois uma dessas talvez possa ser generalizada para se resolver a
parte 2, que eu não consegui fazer.



No mais, não vejo razão para se esconder a própria formação, como o Rafael
parece estar fazendo. Por exemplo, eu sou formado em engenharia elétrica
(ênfase em sistemas) pela PUC-RJ e estudo matemática por conta própria
porque gosto. Hoje em dia estou fazendo dois cursos no IME-USP como
ouvinte - Análise Real e Álgebra III, que consiste de teoria dos corpos e
teoria de Galois.

[]s,
Claudio.


=
Instruções para entrar na lista, sair da lista e usar a lista em
http://www.mat.puc-rio.br/~nicolau/olimp/obm-l.html
=


Re: [obm-l] Problemas com radicais

2004-04-16 Por tôpico Rafael
E, certamente, eu não discordei da sugestão dele, Samuel. Entretanto, é fato
que os usuários de programas como Maple, Mathematica e outros deparam-se com
a notação Sqrt(x) para a raiz quadrada de x. Em programação, ela também é
utilizada. Cbrt(x) é bem menos utilizada, mas se você tiver por hábito ler
mensagens de listas de língua inglesa, verá que o uso é corrente. Por isso,
aquela minha sugestão, pois "sqr3" me pareceu um pouco estranha. Mas é claro
que você não precisa concordar comigo...

Sobre a sua pergunta final, não sei qual foi o seu intento ao realizá-la,
não sei a que mensagens você se refere, mas ela é off-topic para esta lista.


Abraços,

Rafael de A. Sampaio




- Original Message -
From: "Samuel Siqueira" <[EMAIL PROTECTED]>
To: <[EMAIL PROTECTED]>
Sent: Friday, April 16, 2004 4:24 AM
Subject: Re: [obm-l] Problemas com radicais



Rafael,

   creio q o objetivo do Johann não era questionar sua utilização d sqrt e
cbrt, mas d sugerir aquelas outras notações q poderiam propiciar, talvez,
alguma maneira "intuitiva" d resolver o problema. Tb li outras msgs suas,
estou curioso, qual sua formação???

Abraço,

Samuel Siqueira

=
Instruções para entrar na lista, sair da lista e usar a lista em
http://www.mat.puc-rio.br/~nicolau/olimp/obm-l.html
=


Re: [obm-l] Problemas com radicais

2004-04-16 Por tôpico Rafael
Caro "J.P.G.L. Dirichlet",

As notações Sqrt(x) e Cbrt(x) garanto a você que não foram inventadas por
mim... E, sinceramente, não creio que se usem freqüentemente a raiz quarta
ou vigésima tanto quanto ocorre com a quadrada ou a cúbica. Se elas merecem
uma notação à parte? Concordo com o comentário feito pelo Prof. Morgado.





- Original Message -
From: Johann Peter Gustav Lejeune Dirichlet
To: [EMAIL PROTECTED]
Sent: Thursday, April 15, 2004 1:25 PM
Subject: Re: [obm-l] Problemas com radicais


Tentem usar uma notaçao mais matematica,como a^(1/2) seria a raiz
quadrada.Isto juda muito pois por exemplo como voce escreveria raiz quarta
ou vigesima?


=
Instruções para entrar na lista, sair da lista e usar a lista em
http://www.mat.puc-rio.br/~nicolau/olimp/obm-l.html
=


Re: [obm-l] Problemas com radicais - CORRIGINDO!!

2004-04-16 Por tôpico Rafael
Caro Rogério,

Eu não consegui entender o que você não entendeu: qual seria o objetivo de
um comentário a não ser emitir uma opinião que pode ou não ter algum
fundamento? Não me queira mal, por favor. Você nem sequer precisaria ter
mutilado o meu minúsculo comentário para comentá-lo...
Não, você não escreveu que 'a' e 'b' deveriam ser distintos e, em momento
algum, disse que o havia feito. Salientiei, e espero que você tenha
compreendido, que o trecho escrito por você estava entre aspas.
Sim, você me deu um contra-exemplo sobre o qual eu não havia pensado e que
eu encaminharei para o autor do livro que escreveu esses absurdos.

Tudo esclarecido? Espero que sim.


Obrigado,

Rafael de A. Sampaio




- Original Message -
From: "Rogério Moraes de Carvalho" <[EMAIL PROTECTED]>
To: <[EMAIL PROTECTED]>
Sent: Thursday, April 15, 2004 12:36 PM
Subject: RE: [obm-l] Problemas com radicais - CORRIGINDO!!


Ola Rafael,

Eu realmente nao consegui entender o objetivo dos seus comentarios.
De qualquer modo, eu estou comentando-os parte por parte.


Seu comentario:
---
"Se a e b são racionais distintos, então a^2 é racional e a^2 - b é
racional.".

Meu comentario:
---
No texto que eu escrevi, eu nao afirmei em momento algum que a e b devem ser
distintos, mas simplesmente racionais. A sua conclusao sobre a^2 e a^2 - b
serem racionais e' obvia, mesmo que a e b sejam racionais iguais. Isto e'
consequencia da propriedade de fechamento das operacoes de adicao e
multiplicacao do conjunto dos numeros racionais.



Seu comentario:
---
"Ora, se a^2 - b for racional, transformar-se-á sqrt[a +- sqrt(b)] numa soma
ou diferença de radicais duplos, pois sqrt(a^2 - b) será
irracional."

Meu comentario:
---
Esta sua conclusao nao tem o menor embasamento teorico. De qualquer modo,
segue um contra-exemplo bem simples que comprova que a sua conclusao e'
falsa. Suponha a = 5/2 (racional) e b = 4 (racional), entao teremos a^2 - b
= (5/2)^2 - 4 = 25/4 - 4 = 9/4. Sendo assim, sqrt(a^2 - b) = sqrt(9/4) =
3/2, que e' racional. Portanto, a sua conclusao de que sqrt(a^2 - b) será
irracional esta' errada.



Seu comentario:
---
"Dessa forma, sqrt(a^2 - b) deve ser um número inteiro
não-negativo, ou ainda, natural. Por isso: a, b, sqrt(a^2 - b) são
*naturais*, com [a +- sqrt(b)] real positivo."

Meu comentario:
---
Esta conclusao tambem nao tem o menor embasamento teorico. A reducao de
radicais duplos em radicais simples nao exige que a, b e sqrt(a^2 - b) sejam
naturais. Vamos a um exemplo de reducao de radicais duplos em radicais
simples em que a, b e sqrt(a^2 - b) sao racionais nao inteiros.
No radical duplo sqrt(5/3 + sqrt(7/3)), temos a = 5/3, b = 7/3 e sqrt(a^2 -
b) = sqrt[(5/3)^2 - 7/3] = sqrt(25/9 - 7/3) = sqrt[(25 - 21)/9] = sqrt(4/9)
= 2/3.
Sendo assim, podemos converter o radical duplo para radical simples, como
segue:
sqrt(5/3 + sqrt(7/3)) = sqrt[(5/3 + 2/3) / 2] + sqrt[(5/3 - 2/3) / 2]
sqrt(5/3 + sqrt(7/3)) = sqrt(7/6) + sqrt(1/2)


De qualquer modo, eu agradeco pela sua atencao.

Abracos,

Rogério Moraes de Carvalho



=
Instruções para entrar na lista, sair da lista e usar a lista em
http://www.mat.puc-rio.br/~nicolau/olimp/obm-l.html
=


Re: [obm-l] Problemas com radicais - CORRIGINDO!!

2004-04-15 Por tôpico Rafael
RogÃrio,

Farei apenas um comentÃrio sobre as condiÃÃes de reduÃÃo dos radicais duplos
a radicais simples. Vocà escreveu: "Dada a expressÃo com radicais duplos â(a
+ âb), com a e b racionais, âb irracional e a + âb positivo, (...)"

Se a e b sÃo racionais distintos, entÃo a^2 Ã racional e a^2 - b Ã
racional. Ora, se a^2 - b for racional, transformar-se-Ã sqrt[a +- sqrt(b)]
numa soma ou diferenÃa de radicais duplos, pois sqrt(a^2 - b) serÃ
irracional. Dessa forma, sqrt(a^2 - b) deve ser um nÃmero inteiro
nÃo-negativo, ou ainda, natural. Por isso: a, b, sqrt(a^2 - b) sÃo
*naturais*, com [a +- sqrt(b)] real positivo.


AbraÃos,

Rafael de A. Sampaio





- Original Message -
From: "RogÃrio Moraes de Carvalho" <[EMAIL PROTECTED]>
To: <[EMAIL PROTECTED]>
Sent: Thursday, April 15, 2004 8:05 AM
Subject: RE: [obm-l] Problemas com radicais - CORRIGINDO!!


Olà Daniel,

Muitos dos problemas que envolvem expressÃes com radicais duplos podem ser
resolvidos facilmente quando sÃo realizadas as reduÃÃes para expressÃes com
radicais simples equivalentes. Existe uma fÃrmula para a reduÃÃo, mas o
importante à entender como deduzi-la, pois o raciocÃnio à muito simples.

ReduÃÃo de radicais duplos em radicais simples equivalentes
---
Dada a expressÃo com radicais duplos â(a + âb), com a e b racionais, âb
irracional e a + âb positivo, queremos encontrar x1 e x2 racionais positivos
tais que: â(a + âb) = âx1 + âx2.

Observe que de acordo com as condiÃÃes dadas, ambos os membros da igualdade
sÃo positivos. Portanto, a fim de eliminar o radical duplo do primeiro
membro da igualdade, podemos elevar ambos os membros ao quadrado garantindo
que a volta continua vÃlida.
[â(a + âb)]Â = (âx1 + âx2)Â
a + âb = x1 + 2âx1âx2 + x2
a + âb = (x1 + x2) + â(4.x1.x2)

Sendo a, b, x1 e x2 racionais e âb irracional, a igualdade somente vai ser
verdadeira se tivermos:
x1 + x2 = a
4.x1.x2 = b <=> x1.x2 = b/4

Portanto, x1 e x2 sÃo raÃzes da seguinte equaÃÃo quadrÃtica:
x - (x1 + x2)x + x1.x2 = 0 <=> x - ax + b/4 = 0

Calculando o discriminante, encontramos:
Π= (-a) - 4.1.(b/4) <=> Π= a - b

Sendo assim, a nossa expressÃo somente poderà ser reduzida a radicais
simples se o discriminante (a - b) for um quadrado de um racional. Se esta
condiÃÃo for satisfeita, teremos:
x1 = [-(-a) + â(a - b)] / 2 = [a + â(a - b)] / 2
x2 = [-(-a) - â(a - b)] / 2 = [a - â(a - b)] / 2
Ou vice-versa.

ConclusÃo:
A expressÃo com radicais duplos â(a + âb), com a e b racionais, âb
irracional e a + âb positivo, pode ser transformada em uma expressÃo com
radicais simples quando a - b for um quadrado de um racional. A
transformaÃÃo à dada pela seguinte fÃrmula:
â(a + âb) = â{[a + â(a - b)] / 2} + â{[a - â(a - b)] / 2}

Analogamente, podemos demonstrar que a expressÃo com radicais duplos
â(a - âb), com a e b racionais, âb irracional e a - âb positivo, pode ser
transformada em uma expressÃo com radicais simples quando a - b for um
quadrado de um racional. A transformaÃÃo à dada pela seguinte fÃrmula:
â(a - âb) = â{[a + â(a - b)] / 2} - â{[a - â(a - b)] / 2}


ResoluÃÃo do problema proposto:
---
Simplifique a expressÃo:
(2 + â3) / [â2 + â(2 + â3)] + (2 - â3) / [â2 - â(2 - â3)]

Vamos verificar se à possÃvel reduzir as expressÃes com radicais duplos para
expressÃes com radicais simples.
Na expressÃo â(2 + â3), temos a = 2 e b = 3. Como a - b = 4 - 3 = 1, que Ã
o quadrado de um racional (1 = 1Â), a transformaÃÃo à possÃvel.
â(2 + â3) = â[(2 + 1) / 2] + â[(2 - 1) / 2] = â(3/2) + â(1/2) = â3/â2 
+ 1/â2
Analogamente, teremos:
â(2 - â3) = â3/â2 - 1/â2

Logo:
(2 + â3) / [â2 + â(2 + â3)] + (2 - â3) / [â2 - â(2 - â3)] =
= (2 + â3) / [â2 + (â3/â2 + 1/â2)] + (2 - â3) / [â2 - (â3/â2 - 
1/â2)] =
= (2 + â3) / [(2 + â3 + 1)/â2] + (2 - â3) / [(2 - â3 + 1)/â2] =
= â2(2 + â3) / (3 + â3) + â2(2 - â3) / (3 - â3) =
= [â2(2 + â3)(3 - â3) + â2(2 - â3)(3 + â3)] / [(3 + â3) (3 - â3)] =
= [â2(6 - 2â3 + 3â3 - 3) + â2(6 + 2â3 -3â3 - 3)] / (9 - 3) =
= â2[(3 + â3) + (3 - â3)] / 6 = 6â2 / 6 = â2

Portanto, a expressÃo simplificada à igual a â2.

Atenciosamente,

RogÃrio Moraes de Carvalho

=
Instruções para entrar na lista, sair da lista e usar a lista em
http://www.mat.puc-rio.br/~nicolau/olimp/obm-l.html
=


Re: [obm-l] Problemas com radicais

2004-04-15 Por tôpico Rafael
Daniel,

Como sugestão, utilize cbrt(x) para a raiz cúbica de x. Veja que sqrt(x) vem
de "square root" e, semelhantemente, cbrt(x), de "cubic root".

Novamente, para realmente simplificar, estou enviando a solução anexada à
mensagem. Você encontrará a motivação das manipulações que fiz se pensar no
problema de trás para frente, isto é, elevando os dois membros ao cubo, que
você encontraria? Observe isso e faça o caminho de volta.


Abraços,

Rafael de A. Sampaio





- Original Message -
From: "Daniel Silva Braz" <[EMAIL PROTECTED]>
To: <[EMAIL PROTECTED]>
Sent: Wednesday, April 14, 2004 11:09 PM
Subject: [obm-l] Problemas com radicais


1) Simplifique:

sqr((2 + sqr(3)) / (2 - sqr(3))) + sqr((2 - sqr(3)) /
(2 + sqr(3)))

2) Mostre que sqr3(9(sqr3(2) - 1)) = 1 - sqr3(2) +
sqr3(4)

Obs: sqr = raíz quadrada e sqr3 = raíz cúbica

Daniel Silva Braz


radicais2.gif
Description: Binary data


Re: [obm-l] trigonometria

2004-04-14 Por tôpico Rafael
Guilherme,

Antes de conversarmos sobre o seu problema, vamos pensar um pouco,
esquecendo os radianos por agora.

Certamente, você sabe que Pi rad equivale a 180°. Se você tivesse que
encontrar a primeira determinação no ciclo de 450°, como você faria? Ah, é
óbvio que, por a circunferência ter 360°, você dividiria 450° por 360°.

Vamos fazer isso:

   450°   |  360°
- 360°   |  1
 |
90°|

Que descobrimos? Que 450° corresponde a uma volta completa (360°) acrescida
de 90°. Esses 90° são precisamente a primeira determinação no ciclo.

Agora, um outro exemplo, usando o mesmo raciocínio para radianos.

Suponhamos o arco de 63pi/5 rad. Vamos agir da mesma forma, dividindo por
2*Pi rad (= 2*180° = 360°):

   63 Pi / 5   |   2 Pi = 10 Pi / 5
- 60 Pi / 5|   6
 |
 3 Pi / 5   |

Que descobrimos?

Que 63 Pi / 5 rad corresponde a 6 voltas completas mais 3 Pi / 5 rad.

O raciocínio é bastante intuitivo, espero que tenha ficado claro.


Abraços,

Rafael de A. Sampaio


Obs.: Não foi por acaso que, no primeiro exemplo, 450° = 5Pi/2 rad...




- Original Message -
From: Guilherme Teles
To: [EMAIL PROTECTED]
Sent: Tuesday, April 13, 2004 8:50 PM
Subject: [obm-l] trigonometria


alguem pode me dar uma ajuda sobre arcos congruentes.
Como achar a imagem no ciclo de 5pi/2 e 63pi/5
existem formulas ?





=
Instruções para entrar na lista, sair da lista e usar a lista em
http://www.mat.puc-rio.br/~nicolau/olimp/obm-l.html
=


Re: [obm-l] duvida basica: mdc e mmc, conjunto

2004-04-14 Por tôpico Rafael
Em relação à primeira pergunta, leia:

http://www.mat.puc-rio.br/~nicolau/olimp/obm-l.200403/msg00252.html

É uma demonstração que o Cláudio fez no mês passado.


Sobre a segunda pergunta, imagino que você tenha querido dizer:

n(A U B) = n(A) + n(B) - n(A inter B)

Não sei qual é o rigor que o livro exige para a demonstração, mas certamente
algo válido é o que se observa dos diagramas de Venn, que, de fato, são
bastante intuitivos.


Abraços,

Rafael de A. Sampaio





- Original Message -
From: <[EMAIL PROTECTED]>
To: <[EMAIL PROTECTED]>
Sent: Wednesday, April 14, 2004 12:57 AM
Subject: [obm-l] duvida basica: mdc e mmc, conjunto


alguem sabe me dizer pq mdc(a,b) x mmc(a,b) = a x b?

existe alguma explicacao/demonstracao?

agradeco desde jah...

e em

n(a) + n(b) = n(a)+ n(b) - n(a inter b)

nos conjuntos, eu consigo o entender pq (os elementos da intersecao sao
somados duas vezes) intuitivamente. Porem eu estou estudando por um livro
aki q pede pra provar. Alguem poderia me ajudar?

valew

=
Instruções para entrar na lista, sair da lista e usar a lista em
http://www.mat.puc-rio.br/~nicolau/olimp/obm-l.html
=


[obm-l] Re: [obm-l] dúvida

2004-04-14 Por tôpico Rafael
Eu sei que muitas sugestões não são atendidas, mas...

Por exemplo, o Niski já deve estar com os dedos trêmulos de tanto escrever
que algumas mensagens teriam uma recepção melhor em outras listas e/ou
sites. Também já foi sugerido que o assunto dos e-mails fosse melhor
definido, o que facilita e muito a leitura. E, além de tudo isso, que se use
uma notação o mais possível clara, embora você tenha explicado o que queria
dizer. Enfim, não quero parecer pouco cordial repetindo isso e, de fato,
espero não estar sendo.


Em relação aos logaritmos, a minha sugestão é que você use:

logaritmo de x na base y <==> log_y(x)

ou

logaritmo de x na base y <==> log(x,y)


Para o problema 1, lembre-se de que:
log(a,b) = log(a,c) / log(b,c)   (mudança de base)

Assim: a^(log(log(a))/log(a)) = a^log(log(a),a) = log(a),
pois x^log(y,x) = y.


Reescrevendo o problema 2,

a^[log(b,a)*log(c,b)*log(d,c)] =
= a^[log(b,a)*[log(c,a)/log(b,a)]*[log(d,a)/log(c,a)] =
= a^log(d,a) = d

Na segunda linha, fiz a mudança de todos os logaritmos do expoente para a
mesma base da potência. Na terceira linha, utilizei a mesma última
propriedade mencionada no exercício 1.


Abraços,

Rafael




- Original Message -
From: TSD
To: [EMAIL PROTECTED]
Sent: Wednesday, April 14, 2004 12:30 AM
Subject: [obm-l] dúvida


simplificar :

1) "a" está elevado a tudo isto aí => a^ ([log(loga)]/loga)

2)  a ^ (loga^b.logb^c.logc^d) a base é oque está antes do ^



=
Instruções para entrar na lista, sair da lista e usar a lista em
http://www.mat.puc-rio.br/~nicolau/olimp/obm-l.html
=


[obm-l] Re: [obm-l] Re: [obm-l] Re: [obm-l] Dúvida persistente!!!

2004-04-13 Por tôpico Rafael
Auggy,

Independentemente das contas, a criatividade na construção dos triângulos é
magnífica. Lendo o link, vi que o Cláudio já havia pensado no cálculo da
área por integral e teve uma idéia muito melhor em relação à posição dos
eixos, com origem em B em vez de A.

Enfim, apesar de trabalhoso, é um problema bonito.


Cláudio,

Parabéns por ambas as soluções!


Abraços,

Rafael de A. Sampaio





- Original Message -
From: "Qwert Smith" <[EMAIL PROTECTED]>
To: <[EMAIL PROTECTED]>
Sent: Tuesday, April 13, 2004 5:41 PM
Subject: RE: [obm-l] Re: [obm-l] Re: [obm-l] Dúvida persistente!!!


Sai na geometria mas da umas contas chatas.

na primeira figura:
---
(area em amarelo)  = (area do circulo menor) - 2*(area em verde)

(area em vermelho) = (area do quadrado) -
1/4*{(area do circulo maior) + [(area do quadrado)-(area circulo menor)]} -
(area em verde)
---

na segunda figura:
---
(area em laranja) = (setor circular PBQ) - 2*(triangulo PBO - area em azul)
(area em verde) = (setor circular POQ) - (area em laranja)
--

Como todos os lados do triangulo sao conhecidos (em funcao do lado do
quadrado).  Agulos e areas sao questao de braco.

A descricao da figura 2 vc encontra no link

http://www.mat.puc-rio.br/~nicolau/olimp/obm-l.200310/msg00574.html

em uma menssagem do grande Claudio BUffara que ainda teve paciencia
montruosa de me explicar em off o problema.

Em meu email anterior eu tinha feito confusao e atribuido a mensagem do link
acima a outro fera, o Paulo Santa Rita que mandou uma mensagem sobre "lua
algebrica", que tb vale a pena conferir.
E' muito genio pra keep track.

Valeu Super Buffara!

[]s,
Auggy



=
Instruções para entrar na lista, sair da lista e usar a lista em
http://www.mat.puc-rio.br/~nicolau/olimp/obm-l.html
=


[obm-l] Re: [obm-l] Re: [obm-l] Dúvida persistente!!!

2004-04-12 Por tôpico Rafael
Eu desisto...

Tentei encontrar uma solução simples, como pedia o Eduardo, mas a melhor
forma que vejo agora é calcular, por integral, a área verde e só depois
encontrar a área amarela.

Minha idéia é pôr a circunferência de centro A na origem do sistema de
coordenadas; o lado do quadrado não será mais x, e sim R; a equação da
circunferência citada será x^2 + y^2 = R^2. A circunferência inscrita no
quadrado terá equação: (x-R/2)^2 + (y+R/2)^2 = R^2/4. Os pontos de
intersecção das equações são:

( R*(5 + sqrt(7))/8 ; R*(sqrt(7) - 5)/8 )

e

( R*(5 - sqrt(7))/8 ; -R*(5 + sqrt(7))/8 )


A área S amarela será dada por:

S = Pi * R^2/4 - 2*(Integral[- sqrt(R^2 - x^2)] dx -
- Integral[- R/2 + sqrt(x*R - x^2)] dx)

O intervalo das integrais é [R*(5 - sqrt(7))/8 ; R*(5 + sqrt(7))/8].


Depois de muito trabalho algébrico (deixado para o Mathematica),
voltando de R para x, chegamos à expressãozinha anexada a esta mensagem,
por razões óbvias...

Dá para entender o porquê de a questão ser persistente...


Abraços,

Rafael de A. Sampaio





- Original Message -
From: "Rafael" <[EMAIL PROTECTED]>
To: <[EMAIL PROTECTED]>
Sent: Sunday, April 11, 2004 3:12 AM
Subject: [obm-l] Re: [obm-l] Dúvida persistente!!!


Obrigado pelo elogio à figura, Qwert.

Na verdade, o que tornou a minha solução errada foi não ter somado quatro
vezes a área vermelha, pois cada uma acabou sendo subtraída duas vezes. Pelo
que vejo, descobrindo a área vermelha, teremos a área amarela (que foi a que
pretendi calcular) e a diferença da área do círculo menor (de raio x) com
esta área amarela é, precisamente, a área verde.

Descobrir essa área vermelha é que não me parece muito fácil...






FigColor.gif
Description: Binary data


result.gif
Description: Binary data


[obm-l] Re: [obm-l] Re: [obm-l] Elementos da Circunferência

2004-04-11 Por tôpico Rafael
Túlio,

Eu não sei se você viu a figura anexada, pois A está no arco maior, logo
m(BÂC) = 30°, estando descartada a possibilidade deste ser obtuso.


Um forte abraço,

Rafael de A. Sampaio





- Original Message -
From: "Túlio Beronha" <[EMAIL PROTECTED]>
To: <[EMAIL PROTECTED]>
Sent: Sunday, April 11, 2004 12:29 PM
Subject: [obm-l] Re: [obm-l] Elementos da Circunferência


Seja O o centro da referida circunferência.
O ponto A pode estar sobre qualquer um dos arcos detderminados pela corda
BC.
O triângulo BCO é equilátero. Se A está no arco maior o teorema do ângulo
inscrito nos diz que
BAC mede 30 graus. Se, por outro lado, o ponto A está sobre o arco menor o
ângulo BAC mede 150 graus.
O teorema dos senos também implica essas duas possibilidades, senA=1/2 e A
ângulo interno de um
triângulo ou A mede 30 graus, ou A mede 150 graus.
Saludos
Túlio Beronha.


=
Instruções para entrar na lista, sair da lista e usar a lista em
http://www.mat.puc-rio.br/~nicolau/olimp/obm-l.html
=


[obm-l] Re: [obm-l] Dúvida persistente!!!

2004-04-10 Por tôpico Rafael
Obrigado pelo elogio à figura, Qwert.

Na verdade, o que tornou a minha solução errada foi não ter somado quatro
vezes a área vermelha, pois cada uma acabou sendo subtraída duas vezes. Pelo
que vejo, descobrindo a área vermelha, teremos a área amarela (que foi a que
pretendi calcular) e a diferença da área do círculo menor (de raio x) com
esta área amarela é, precisamente, a área verde.

Descobrir essa área vermelha é que não me parece muito fácil...



- Original Message -
From: "Qwert Smith" <[EMAIL PROTECTED]>
To: <[EMAIL PROTECTED]>
Sent: Sunday, April 11, 2004 12:34 AM
Subject: RE: [obm-l] Dúvida persistente!!!


Vou pegar carona na figura bonitinha que o Rafael fez.

A muito pouco tempo  atras o Carlos mandou uma questao na qual ele queria
saber a area em vermelho.  Vc quer saber a area em amarelo.  Veja que ambos
os problemas podem se resumir em descobrir a area em verde.  Essa area da
lua ja foi tratada na lista e tem uma mensagem muito boa do Paulo Santa
Rita.  Procure no google por 'area da lua'.  Tb vale a pena procurar pelo
'problema do cavalo' (as vezes 'problema do burro') a que o Paulo faz
referencia.



=
Instruções para entrar na lista, sair da lista e usar a lista em
http://www.mat.puc-rio.br/~nicolau/olimp/obm-l.html
=


[obm-l] Re: [obm-l] Re: [obm-l] Dúvida persistente!!!

2004-04-10 Por tôpico Rafael
Esta resolução está errada... :-(

Vou tentar consertar e reenvio depois.



- Original Message -
From: "Rafael" <[EMAIL PROTECTED]>
To: <[EMAIL PROTECTED]>
Sent: Saturday, April 10, 2004 9:50 PM
Subject: [obm-l] Re: [obm-l] Dúvida persistente!!!


Eduardo,

Esse exercício é facilitado se você fizer algumas construções.

Primeiramente, vamos subtrair a área de um setor de 90° e raio x da área do
quadrado ABCD:

S1 = x^2 - (Pi * x^2)/4 = x^2 * (1 - Pi/4)

Depois disso, ligue o centro da circunferência inscrita no quadrado ao ponto
médio de dois lados adjacentes do quadrado; construiremos um quadrado de
lado x/2. Da área deste quadrado subtraímos a área de um setor de 90° e raio
x/2:

S2 = (x/2)^2 - (Pi * (x/2)^2)/4 = x^2  * (1 - Pi/4) / 4

Pronto! A área que procuramos é:

S = x^2 - 2 * S1 - 2 * S2 = x^2 - 2 * (S1 - S2)
S = x^2 - 2 * (3/4 * x^2 * (1 - Pi/4))
S = x^2 - 3/2 * x^2 * (1 - Pi/4)
S = x^2 * (1 - 3/2 * (1 - Pi/4)
S = x^2 * (1 - 3/2 + 3*Pi/8)

S = x^2 * (3*Pi - 4) / 8


Abraços,

Rafael de A. Sampaio





- Original Message -
From: "Eduardo de Melo Beltrão" <[EMAIL PROTECTED]>
To: <[EMAIL PROTECTED]>
Sent: Saturday, April 10, 2004 7:02 PM
Subject: [obm-l] Dúvida persistente!!!


Olá pessoal,
Tenho uma dúvida que já perdura por anos. Gostaria de compartilhar com
vocês, e se a resposta já foi lançada na lista, gostaria apenas que
indicassem o caminho para eu poder analisar. Desde já agradeço.
Eduardo Beltrão

Num quadrado ABCD de lado x está inscrita uma circunferência L1. Os vértices
opostos A e C do quadrado são centros das circunferências L2 e L3, de raios
igual ao lado do mesmo. Determinar a área da região formada pela interseção
de L1, L2
e L3 em função de x. (Tente usar apenas conhecimentos de geometria plana).



=
Instruções para entrar na lista, sair da lista e usar a lista em
http://www.mat.puc-rio.br/~nicolau/olimp/obm-l.html
=


[obm-l] Re: [obm-l] Dúvida persistente!!!

2004-04-10 Por tôpico Rafael
Eduardo,

Esse exercício é facilitado se você fizer algumas construções.

Primeiramente, vamos subtrair a área de um setor de 90° e raio x da área do
quadrado ABCD:

S1 = x^2 - (Pi * x^2)/4 = x^2 * (1 - Pi/4)

Depois disso, ligue o centro da circunferência inscrita no quadrado ao ponto
médio de dois lados adjacentes do quadrado; construiremos um quadrado de
lado x/2. Da área deste quadrado subtraímos a área de um setor de 90° e raio
x/2:

S2 = (x/2)^2 - (Pi * (x/2)^2)/4 = x^2  * (1 - Pi/4) / 4

Pronto! A área que procuramos é:

S = x^2 - 2 * S1 - 2 * S2 = x^2 - 2 * (S1 - S2)
S = x^2 - 2 * (3/4 * x^2 * (1 - Pi/4))
S = x^2 - 3/2 * x^2 * (1 - Pi/4)
S = x^2 * (1 - 3/2 * (1 - Pi/4)
S = x^2 * (1 - 3/2 + 3*Pi/8)

S = x^2 * (3*Pi - 4) / 8


Abraços,

Rafael de A. Sampaio





- Original Message -
From: "Eduardo de Melo Beltrão" <[EMAIL PROTECTED]>
To: <[EMAIL PROTECTED]>
Sent: Saturday, April 10, 2004 7:02 PM
Subject: [obm-l] Dúvida persistente!!!


Olá pessoal,
Tenho uma dúvida que já perdura por anos. Gostaria de compartilhar com
vocês, e se a resposta já foi lançada na lista, gostaria apenas que
indicassem o caminho para eu poder analisar. Desde já agradeço.
Eduardo Beltrão

Num quadrado ABCD de lado x está inscrita uma circunferência L1. Os vértices
opostos A e C do quadrado são centros das circunferências L2 e L3, de raios
igual ao lado do mesmo. Determinar a área da região formada pela interseção
de L1, L2
e L3 em função de x. (Tente usar apenas conhecimentos de geometria plana).



Fig.gif
Description: Binary data


[obm-l] Re: [obm-l] Re:[obm-l] Elementos da Circunferência

2004-04-10 Por tôpico Rafael
Osvaldo,


Uma das fórmulas que você citou não está correta.

S = AB*AC*BC / 4R = AB*AC*sen(BÂC) / 2

Simplificando como você fez, chegamos a:

BÂC = arc sen 1/2 = 30°.


Não é preciso usar qualquer calculadora...


Abraços,

Rafael de A. Sampaio





- Original Message -
From: "Osvaldo" <[EMAIL PROTECTED]>
To: "obm-l" <[EMAIL PROTECTED]>
Sent: Saturday, April 10, 2004 5:19 PM
Subject: [obm-l] Re:[obm-l] Elementos da Circunferência


Temos AB=r => temos q a area S do triang. ABC é dada por

S=AB.AC.BC/4r
e S=AB.AC.sen(BÂC)

Daí, AB.AC.BC/4r=AB.AC.sen(BÂC)=> como r=BC => sen(BÂC)
=1/4
logo BÂC= arc sen(0.25)
é só pegar uma calculadora.





> Na figura [Ver Anexo] Abaixo , o lado BC do triângulo
é congruente ao raio
> da circunferência .
> Qual a medida do ângulo BÂC?


=
Instruções para entrar na lista, sair da lista e usar a lista em
http://www.mat.puc-rio.br/~nicolau/olimp/obm-l.html
=


[obm-l] Re: [obm-l] Elementos da Circunferência

2004-04-10 Por tôpico Rafael
Pedro,

Pela lei dos senos:

BC / sen(BÂC) = 2R
sen(BÂC) = R / (2R) = 1/2

Portanto, m(BÂC) = 30°.


Abraços,

Rafael de A. Sampaio




- Original Message -
From: "pedro rajão" <[EMAIL PROTECTED]>
To: <[EMAIL PROTECTED]>
Sent: Saturday, April 10, 2004 4:41 PM
Subject: [obm-l] Elementos da Circunferência


Na figura [Ver Anexo] Abaixo , o lado BC do triângulo é congruente ao raio
da circunferência .
Qual a medida do ângulo BÂC?


=
Instruções para entrar na lista, sair da lista e usar a lista em
http://www.mat.puc-rio.br/~nicolau/olimp/obm-l.html
=


Re: [obm-l] problemas de relogios

2004-04-09 Por tôpico Rafael
Para os dois problemas, o raciocínio é o mesmo. Vamos pensar:

Queremos que os ponteiros estejam afastados entre si por 90°. O ponteiro das
horas, por hora, anda 30° e o dos minutos, 360°. Assim, quando o das horas
tiver andado x graus, queremos que o dos minutos esteja em 90°+x.

30° - 360°
  x  -  90° + x

x = 90/11°


Quanto será que isso vale em minutos? Agora, é fácil:

  360°     60 min
90° + 90/11 °     y

y = 180/11 min = 16min 21,8s  (aprox.)


Sabemos, então, que a primeira vez que os ponteiros fazem 90° entre si é ao
meio-dia, dezesseis minutos e alguns segundos. Quando acontecerá de novo?
Para todo y*k, com k ímpar positivo.

k = 3 ==> 180 * 3 / 11 min = 49min 5,5s (não convém)

k = 5 ==> 180 * 5 / 11 min = 1h 21min 49,1s


Portanto, o primeiro instante, após uma hora, ocorre às 13h 21min 49,1s.


O problema 2 se resolve se admitirmos k par positivo e não múltiplo de 4
(quando os ponteiros seriam coincidentes):

k = 6 ==> 180 * 6 / 11 min = 1h 38min 120/11s

k = 10 ==> 180 * 10 / 11 min = 2h 43min 420/11s

k = 14 ==> 180 * 14 / 11 min = 3h 49min 60/11s



Abraços,

Rafael de A. Sampaio





- Original Message -
From: [EMAIL PROTECTED]
To: [EMAIL PROTECTED]
Sent: Friday, April 09, 2004 2:18 AM
Subject: [obm-l] problemas de relogios


Ola pessoal,

Como resolver estes ?

1) Em que instante depois das 13:00 horas ficam os ponteiros de um relogio
em angulo reto ?

2) A que horas os ponteiros das horas e dos minutos de um relogio estarao no
prolongamento, um do outro, logo depois de estarem marcando precisamente 3
horas ?

ps: Coloquei o gabarito la embaixo para quem quiser resolver e comparar
depois.


gabarito


1) 13h 21min 49,1s
2) 3h 49min 60/11s


=
Instruções para entrar na lista, sair da lista e usar a lista em
http://www.mat.puc-rio.br/~nicolau/olimp/obm-l.html
=


Re: [obm-l] algumas duvidas de PA

2004-04-08 Por tôpico Rafael
Qwert,

Como sempre, você tem saídas rápidas para os problemas! ;-)

De fato, muitos cálculos que eu mostrei eram dispensáveis. Na verdade, eles
estavam lá mais para conduzir o raciocínio do que por necessidade. A sua
solução, certamente, é mais rápida para quando já se conhecem as
"engrenagens" do problema. Gostei, parabéns!


Um forte abraço,

Rafael de A. Sampaio




- Original Message -
From: "Qwert Smith" <[EMAIL PROTECTED]>
To: <[EMAIL PROTECTED]>
Sent: Wednesday, April 07, 2004 3:00 PM
Subject: Re: [obm-l] algumas duvidas de PA


So vou adicionar meus 2 centavos ja que ja esta resolvido... concordo com o
Rafael que o problema 3 e o mais (unico remotamente?) interessante, mas acho
interessante justamente pq nao precisa de todos os calculos

PA1 razao 3 tamanho 100
PA2 razao 4 tamanho 100

Sabemos que o primeiro termo comum e o 3o termo ( nao importa se e 11, 100,
coelinho da pascoa, etc)
O Rafael conclui com razao (trocadilho, por favor), que os termos comuns tem
12 como razao entre eles.  Isso acontece a cada 3 termos na PA2 e a cada 4
termos na PA1... hmmm...perai
na PA1 tenho tenho mais 97 termos... mas como so interessa de 4 em 4 so me
interessam 24 (parte inteira 97/4).  Tenho entao 24 termos + o 3o temo ki ja
sabiamos ser comum.  24 + 1 = 25.
Nao precisa identificar nenhum dos termos.  Isso ki eu achei interessante.

P.S.  Logicamente o '11' e importante na determinacao das razoes das 2 PAs.


=
Instruções para entrar na lista, sair da lista e usar a lista em
http://www.mat.puc-rio.br/~nicolau/olimp/obm-l.html
=


Re: [obm-l] PG

2004-04-07 Por tôpico Rafael
Guilherme,

Não se preocupe, nenhum problema é bobo até que você saiba como resolvê-lo.
Certa vez, comentei algo semelhante sobre os problemas chamados de triviais
se distinguirem dos não-trivais somente pelo fato destes nunca terem sido
resolvidos por alguém... ;-)

Vamos aos exercícios. O modo de resolução do primeiro usa um artifício bem
conhecido, que é representar três termos de uma P.G. (ou P.A.) em função do
termo do meio, assim:

P.G.:  a/q, a, a*q
P.A.:  a - r, a, a + r

Depois de conhecido esse artifício, o que nos resta são as contas:

a/q + a + aq = 21/8   (I)
(a/q)^2 + a^2 + (a*q)^2 = 189/64   (II)

Elevando (I) ao quadrado e substituindo (II):

189/64 + 2(a^2/q + a^2 + a^2q) = 441/64
2a(a/q + a + aq) = (441-189)/64 = 63/16
2a(21/8) = 63/16
a*21/4 = 63/16
a = 3/4

Voltando 'a' em (I):

(3/4)/q + 3/4 + (3/4)q = 21/8
3/(4q) + 3q/4 = (21-6)/8 = 15/8
3 + 3q^2 = (15*4q)/8 = 15q/2
2q^2 - 5q + 2 = 0

D = 25 - 4*2*2 = 9
q = (5 +- 3)/4 ==> q = 1/2 ou q = 2

q = 1/2 ==> (..., 3/2, 3/4, 3/8, ...)  ==> P.G. decrescente e convergente
q = 2 ==> (..., 3/8, 3/4, 3/2, ...)  ==> P.G. crescente


Já o exercício 2 se assemelha muito ao exercício 2 de P.A. que você mandou
ontem. Dê uma comparada depois.

a1 + a2 = 12
a3 + a4 = 300

Novamente, colocando os termos em função de a1 e da razão q:

a1 + a1q = 12 <==> a1(1 + q) = 12
a1q^2 + a1q^3 = 300 <==> a1q^2(1 + q) = 300


ATENÇÃO: vou dividir a segunda equação pela primeira, mas tão somente por
saber que a1 é diferente de zero (se fosse zero, a soma dos dois primeiros
termos não poderia ser 12 qualquer que fosse a razão). Também se pode
garantir que (1+q) <> 0, pois se (1+q) = 0, isto é, q = -1, então a soma de
dois termos consecutivos seria nula:

a1*(-1) + a1*(-1)^2 = 0

Sabemos que isso não é verdade do enunciado, então podemos dividir com
tranqüilidade:

q^2 = 25 ==> q = 5 ou q = -5

q = 5 ==> a1 = 2 ==> (2, 10, 20, 40, ...)  P.G. crescente
q = -5 ==> a1 = -3 ==> (-3, 15, -75, 375, ...)  P.G. alternante


Abraços,

Rafael de A. Sampaio





- Original Message -
From: Guilherme Teles
To: [EMAIL PROTECTED]
Sent: Wednesday, April 07, 2004 8:07 PM
Subject: [obm-l] PG


1 - Determine tres numeros reais em PG de modo que sua soma seja 21/8 e a
soma de seus quadrados seja 189/64

2 - Obtenha a PG de quatro elementos em que a soma dos dois primeiros é 12 e
a soma dos dois ultimos é 300

Caros colegas de lista, sei que parecem bobos, mas faz 3 anos que não toco
em materia de 2 grau.

Fico agradecido e humildemente agradeço de coração a colaboração e atenção
que todos tem cedido.

Sds,
Guilherme Teles
Belem - PA



=
Instruções para entrar na lista, sair da lista e usar a lista em
http://www.mat.puc-rio.br/~nicolau/olimp/obm-l.html
=


Re: [obm-l] algumas duvidas de PA

2004-04-07 Por tôpico Rafael
Guilherme,

Para o problema 1, observe que os extremos do intervalo 100 e 1000 não são
múltiplos de 11. Mas quais serão os mais próximos?

11*9 = 99, que não pertence ao intervalo
11*10 = 110, que pertence no intervalo

Então, já sabemos que o primeiro múltiplo de 11 no intervalo é 110.

Analogamente:

11*90 = 990, que pertence no intervalo
11*91 = 1001, que não pertence ao intervalo

Pronto, montamos a nossa seqüência:

110, ..., 990

Se 110 é o primeiro termo, qual é a posição de 990?

990 = 110 + (n-1)*11 ==> n = 81

Assim, a_81 = 990 e a P.A. tem 81 termos, isto é, há 81 múltiplos de 11
entre 100 e 1000.


Sobre o problema 2, vamos passar para o "matematiquês":

a_1 + a_2 = 5
a_9 + a_10 = 53

Para que se possa definir bem um termo de P.A. ou de P.G., de que precisamos
saber? Certamente, o primeiro termo e a razão. Então, reescreveremos as
equações anteriores em função deles:

a_1 + a_1 + r = 5
a_1 + 8r + a_1 + 9r = 53

Ou ainda,

2a_1 + r = 5 (I)
2a_1 + 17r = 53(II)

Como queremos saber apenas a razão, eliminamos 2a1, assim:

(II) - (I): 17r - r = 53 - 5 ==> 16r = 48 ==> r = 3


O terceiro problema é o mais interessante dos três. Vamos escrever as
seqüências e calcular o último termo de ambas:

5, 8, 11, ..., 302(302 = 5 + 99*3)

3, 7, 11, ..., 399(399 = 3 + 99*4)

Já sabemos que ambas possuem igualmente o terceiro termo, 11, e que a
segunda seqüência cresce mais rapidamente que a primeira após o terceiro
termo. Assim, vamos procurar qual é o termo da segunda seqüência mais
próximo do último da primeira:

302 = 3 + (n-1)*4 ==> n = 75,75

Vemos que o termo mais próximo é o septuagésimo quinto. Vamos calculá-lo:

a_75 = 3 + 74*4 = 299

Não deve ser por acaso que o nonagésimo nono termo da primeira seqüência é
299 também, pois 302 - 3 = 299. Assim, já encontramos o primeiro e o último
termos que são iguais para ambas seqüências:

11, ..., 299

Agora, vamos pensar:

a_n = 5 + (n - 1)*3  <==>  a_n - 5 = (n - 3)*3

b_n = 3 + (n - 1)*4  <==> b_n - 3 = (n - 1)*4.

Generalizando,

x_m = x_n + (m - n)*r  <==> x_m - x_n = (m - n)*r

Se ainda não parecer claro o que estou pretendendo, lá vai: a diferença
entre dois termos de uma P.A. é um múltiplo da razão; para que os termos das
seqüências se "encontrem" (sejam iguais), eles devem ser múltiplos de uma
mesma razão, no nosso caso, 3 e 4. Humm... múltiplos de um mesmo número...
múltiplo comum! Sim, é isso! Tudo se passa como se dentro da nossa última
seqüência (11, ..., 299) tivéssemos "despejado" uma seqüência de razão 12,
pois mmc(3,4) = 12. Agora, fica fácil. Qual é a posição do último termo 299?

299 = 11 + (n - 1)*12 ==> n = 288/12 + 1 = 25

Logo, 25 termos são iguais para as duas seqüências.

Quais são eles?

11, 23, 35, 47, 59, 71, 83, 95, 107, 119,
131, 143, 155, 167, 179, 191, 203, 215, 227, 239,
251, 263, 275, 287, 299


Tudo bem, o Mathematica deu uma mãozinha... ;-D


Abraços,

Rafael de A. Sampaio





- Original Message -
From: Guilherme Teles
To: [EMAIL PROTECTED]
Sent: Tuesday, April 06, 2004 10:48 PM
Subject: [obm-l] algumas duvidas de PA


1 - Quantos multiplos de 11 existem entre 100 e 1000

2 - Determine a razão de uma PA com dez termos, sabendo que a soma dos dois
primeiros é 5 e a soma dos dois ultimos é 53

3 - As progressões aritmeticas 5, 8, 11,  e 3, 7, 11,  tem 100
numeros cada uma. Determine o numero de termos iguais nas duas progressões




=
Instruções para entrar na lista, sair da lista e usar a lista em
http://www.mat.puc-rio.br/~nicolau/olimp/obm-l.html
=


Re: [obm-l] Geometria

2004-04-07 Por tôpico Rafael
Artur,

Se seguirmos a sua lógica, as figuras são mérito do programa que utilizei, e
não meu... ;-)
Mas brincadeiras à parte, o elogio não foi somente pelos exercícios, mas por
você enxergar a beleza deles também. Isso, ainda mais para quem se diz não
muito bom em Geometria, é algo elogiável, sim!

Realmente, eu me precipitei e errei, P não é único. A sua solução está
mais-que-perfeita, embora eu não me lembre dos porquês de o semiperímetro p
de ARS igualar-se ao segmento AM e de AM = p' - BC. Quais propriedades dos
triângulos justificam isso?

Sobre o segundo problema, na hora em que resolvi, não pensei no conceito de
potência de ponto, mas certamente é um modo muito interessante de se
raciocinar.

Um forte abraço,

Rafael de A. Sampaio




- Original Message -
From: "Artur Costa Steiner" <[EMAIL PROTECTED]>
To: <[EMAIL PROTECTED]>
Sent: Tuesday, April 06, 2004 11:00 PM
Subject: RE: [obm-l] Geometria


Oi Rafael,
Obrigado pelo elogio aos dois problemas, os quais, alias, devem ser
encaminhados a quem os bolou. As figuras que vc fez ficaram excelentes.

Com relacao ao primeiro problema, acho que houve uma interpretacao
equivocada de sua parte. Na realidade, o perimetro de ARS independe da
posicao de P sobre o arco MN, nao eh preciso que RS seja paralelo a BC.
Aproveitando sua figura e lembrando as propriedades dos triangulos, vejamos:
Com relacao ao triangulo ARS, C eh o circulo exinscrito relativo aos lados
AR e AS. Pelas propriedades dos triangulos, o semiperimetro p de ARS
iguala-se ao segmento AM. Assim, p = AM. E AM independe completamente do
ponto P! Mas, indo um pouco mais longe, temos, tambem pelas propriedades dos
triangulos, que AM = p' - BC, sendo p' o semiperimetro de ABC. Logo, o
perimetro de ARS eh 2p = 2p' - 2BC = AB + AC - BC, qualquer que seja o ponto
P.

Com relacao ao segundo problema, a sua solucao estah perfeita. Mas no dia
12/01/1970 (agora todo muito jah percebeu que naum sou extamente um
garoto...), no Maracanan, no Rio, sob um tremendo calor, eu utilizei o
conceito de potencia de um ponto com relacao a um circulo. Conforme vc fez,
2R1 = 2R2 + 2R3 <==> R1 = R2 + R3. Sendo M o ponto em que C2 e C3 se
tangenciam, a potencia de M com relacao a C nos conduz a que (t/2) * (t/2) =
2R1 * 2R2. E prosseguindo como vc fez, chegamos de fato a S = pi* t^2/8.

Um abraco!
Artur


=
Instruções para entrar na lista, sair da lista e usar a lista em
http://www.mat.puc-rio.br/~nicolau/olimp/obm-l.html
=


Re: [obm-l] Somatorios de k^6 e de k^8

2004-04-07 Por tôpico Rafael
Perdoe-me, Nicolau, por não ter respondido tão imediatamente à sua mensagem.
Muito obrigado pelos seus comentários e, agora, pelos do Gugu e do Angelo.

Talvez, a minha falta de perícia no assunto tenha me feito compreender algo
errado do que li, mas pode ser que o autor não tenha sido tão feliz na
explicação como vocês foram.

Vejam:

http://pessoal.sercomtel.com.br/matematica/superior/zerozero/zero.htm


Obrigado de novo,

Rafael de A. Sampaio




- Original Message -
From: "Nicolau C. Saldanha" <[EMAIL PROTECTED]>
To: <[EMAIL PROTECTED]>
Sent: Tuesday, April 06, 2004 1:44 PM
Subject: Re: [obm-l] Somatorios de k^6 e de k^8


On Tue, Apr 06, 2004 at 01:09:13PM -0300, Carlos Gustavo Tamm de Araujo
Moreira wrote:
> >Você parece estar falando em limites em parte do seu texto. Não é verdade
> >que se lim_{x -> 0} f(x) = 0 e lim_{x -> 0} g(x) = 0 então sempre
> >lim_{x -> 0} ((f(x))^(g(x))) = 1, nem se f e g forem analíticas.
>
> Bem, se f e g sao analiticas nao-constantes numa vizinhanca de 0 e se
anulam
> em 0  (e f e' nao-negativa numa vizinhanca de 0) entao vale
> lim_{x -> 0} ((f(x))^(g(x))) = 1.

Sim, você tem razão. Deve ser isto que o Rafael tinha em mente.

[]s, N.

=
Instruções para entrar na lista, sair da lista e usar a lista em
http://www.mat.puc-rio.br/~nicolau/olimp/obm-l.html
=


Re: [obm-l] Geometria

2004-04-06 Por tôpico Rafael
Artur,

Antes de resolver os problemas que você propôs, devo elogiá-lo pelo bom
gosto, pois eles são realmente muito *bonitinhos*! ;-)

Pela figura 1, se o ponto P estiver mais próximo de M ou N, então RS não
será mais tangente a C.  Assim, P é único para as condições do problema, tal
que o triângulo ARS é semelhante ao ABC (AA~) e 2p/2P = k, sendo 2p o
perímetro do triângulo ARS, 2P o perímetro do triângulo ABC e k a razão de
semelhança. (Espero que isso seja suficiente para mostrar o que o problema
pede.)

Sobre o problema 2 e pela figura 2, sabemos que:

2R1 = 2R2 + 2R3 <==> R1 = R2 + R3

Assim, C1T = R2 - R3.

Ligando C1 a uma das extremidades do segmento t, pelo Teorema de Pitágoras,
teremos:

(R2 - R3)^2 + (t/2)^2 = R1^2 = (R2 + R3)^2
-2 * R2 * R3 + t^2/4 = 2 * R2 * R3
2 * R2 * R3 = t^2/8

A área S será dada por:

S = Pi * R1^2 - Pi * R2^2 - Pi * R3^2
S = Pi * (R1^2 - R2^2 - R3^2)
S = Pi * ((R2 + R3)^2 - R2^2 - R3^2)
S = Pi * 2 * R2 * R3
S = Pi * t^2 / 8


Um forte abraço,

Rafael de A. Sampaio





- Original Message -
From: "Artur Costa Steiner" <[EMAIL PROTECTED]>
To: "OBM" <[EMAIL PROTECTED]>
Sent: Tuesday, April 06, 2004 12:34 PM
Subject: [obm-l] Geometria


Muitos aqui gostam de Geometria. Eu, que nunca foi bom
nela, acho estes problemad do vestibular de 1970, RJ,
muito bonitinhos:

1)Considemos um triangulo ABC e o circulo C nele
inscrito. Sejam M e N os pontos em que C tangencia os
lados AB e AC, respectivamente. A partir de um ponto
P, localizado sobre o arco MN e distinto de M e de N,
tracemos a tangente a C, a qual intersecta os lados AB
e AC nos pontos R e S.  Mostre que o perimetro do
triangulo ARS independe de P.

2) Considere os circulos C1, C2 e C3, tais que C2 e C3
sao interiores a C, tangentes entre si e tangentes a
C. Seja t o segmento situado sobre a tangente comum a
C2 e C3 e delimitado pelas interseccoes desta tangente
com C. Seja S a area da regiao do plano de C interior
a C e exterior a C2 e a C3. Determine S conhecendo t.

Estes problemas sao faceis. Prova: Eu os resolvi no
vestibular.

Artur



2.gif
Description: Binary data


1.gif
Description: Binary data


Re: [obm-l] Quadrilatero ciclico

2004-04-06 Por tôpico Rafael
Caro "J.P.G.L. Dirichlet",

Eu estive pensando sobre o seu problema e provarei um caso particular,
admitindo que o quadrilátero cíclico seja um quadrado e M seja um ponto
qualquer do primeiro quadrante e no interior desse quadrado. A demonstração
para M nos outros quadrantes é análoga, portanto omitirei. No caso mais
geral de um quadrilátero cíclico qualquer, não será mito difícil provar,
talvez um pouco trabalhoso, mas como você ama trigonometria, vai adorar...

Suponhamos que M tenha coordenadas (x;y). Imaginando um quadrado inscrito na
circunferência de equação x^2 + y^2 = r^2, a projeção ortogonal de M em BC
gera F e em AD gera H; o ponto médio de FH será o ponto P(0;y). A projeção
ortogonal de M em AB gera o ponto E e em CD gera o ponto G; o ponto médio de
EG será o ponto Q(x,0). Agora, lembremo-nos de que as diagonais do quadrado
estão contidas nas bissetrizes dos quadrantes pares e ímpares de equações,
respectivamente, x + y = 0 e x - y = 0. A projeção ortogonal de M em BD gera
o ponto L(a,a) e em AC o ponto K(-b,b); para descobrimos tais coordenadas,
levamos em conta que a distância entre os pontos M e L será igual à
distância do ponto M à reta bissetriz dos quadrantes ímpares;
semelhantemente, a distância entre os pontos M e K será igual à distância do
ponto M à reta bissetriz dos quadrantes pares. Assim:

sqrt((x - a)^2 + (y - a)^2) = |x - y|/sqrt(2)

e

sqrt((x + b)^2 + (y - b)^2) = |x + y|/sqrt(2)


Desenvolvendo essas equações e resolvendo para 'a' e 'b', obteremos:

a = (x + y)/2eb = (y - x)/2


Portanto, L((x + y)/2 ; (x + y)/2) e K((x - y)/2 ; (y - x)/2).

O ponto médio de KL será o ponto R(x/2 ; y/2).

Calculando o determinante da matriz correspondente aos pontos P, Q e R:

|  x 0 1 | |  101 |
|  0 y 1 |   =  xy  |  011 |
|  x/2  y/2  1 | | 1/2 1/2  1 |

Pelo teorema de Jacobi, podemos multiplicar a terceira linha por 2 e
subtrair da primeira linha:

  |   0 11  |
xy  |   0 11  |   =  0
  | 1/2  1/2   1  |

Tínhamos, assim, uma das linhas da matriz como combinação linear de outras
duas; sabemos que quando duas filas paralelas são iguais o determinante é
nulo. Logo, se o determinante é nulo, os pontos são colineares, conforme
queríamos demonstrar.

É claro que a solução foi facilitada, pois conhecíamos de antemão as retas
que contêm as diagonais do quadrado, as coordenadas dos pontos P e Q,
enfim... Tudo isso terá de ser esquecido para um quadrilátero cíclico
qualquer e aí está a parte trabalhosa. Ressalto que esta foi apenas uma
sugestão, em que aproveitei para demonstrar um caso particular desse
teorema.


Abraços,

Rafael de A. Sampaio





- Original Message -
From: Johann Peter Gustav Lejeune Dirichlet
To: [EMAIL PROTECTED]
Sent: Monday, March 22, 2004 8:04 PM
Subject: [obm-l] Quadrilatero ciclico


ABCD é um quadrilátero cíclico. M é um ponto qualquer. E, F, G, H, K, L são
as projeções de M em AB, BC, CD, DA, AC, BD, respectivamente. Prove que os
pontos médios de EG, FH, KL são colineares.



=
Instruções para entrar na lista, sair da lista e usar a lista em
http://www.mat.puc-rio.br/~nicolau/olimp/obm-l.html
=


Re: [obm-l] PA

2004-04-05 Por tôpico Rafael
Um pequeno detalhe para a segunda equação:

1/(6-r) + 1/6 + 1/(6+r) = 23/30
1/(6-r) + 1/(6+r) = (23-5)/30 = 3/5
(6+r+6-r)/(36-r^2) = 3/5
20 = 36 - r^2
r = 4  ou  r = -4


Para r = 4, a seqüência é: ..., 2, 6, 10, ...
(progressão aritmética crescente)

Para r = -4, a seqüência é: ... ,10, 6, 2, ...
(progressão aritmética decrescente)



Abraços,

Rafael de A. Sampaio





- Original Message -
From: "David M. Cardoso" <[EMAIL PROTECTED]>
To: <[EMAIL PROTECTED]>
Sent: Monday, April 05, 2004 11:33 PM
Subject: RES: [obm-l] PA




(a - r) + a + (a + r) = 18
3a = 18
a = 6

1/(6-r) + 1/6 + 1/(6+r) = 23/30

[... contas ...]

r = 4

S = { 2, 6, 10 }


=
Instruções para entrar na lista, sair da lista e usar a lista em
http://www.mat.puc-rio.br/~nicolau/olimp/obm-l.html
=


[obm-l] Re: [obm-l] Re: [obm-l] RES: [obm-l] Polígonos

2004-04-05 Por tôpico Rafael
Creio que a equação que o Guilherme pretendia era:

2*130° + (n-2)*128° = (n-2)*180°
260 + 128n - 256 = 180n - 360
52n = 364

Logo, n = 7.



- Original Message -
From: "Thor" <[EMAIL PROTECTED]>
To: <[EMAIL PROTECTED]>
Sent: Monday, April 05, 2004 11:20 PM
Subject: [obm-l] Re: [obm-l] RES: [obm-l] Polígonos

Essa  soluçao do jeito que tá 260 = 0, simplificando os dois lados,
(n-2)180 = 180( n-2)


- Original Message -
From: Guilherme <[EMAIL PROTECTED]>
To: <[EMAIL PROTECTED]>
Sent: Monday, April 05, 2004 8:36 PM
Subject: [obm-l] RES: [obm-l] Polígonos


> Olá,
>
> A soma dos ângulos internos é:
>
> 260 + (n-2).180 = 180(n-2)
> Logo, n = 7
>
> Um abração,
>
> Guilherme Marques.
>




> -Mensagem original-
> De: [EMAIL PROTECTED] [mailto:[EMAIL PROTECTED] Em
> nome de aryqueirozq
> Enviada em: segunda-feira, 5 de abril de 2004 20:17
> Para: [EMAIL PROTECTED]
> Assunto: [obm-l] Polígonos
>
>
>
>
> Dois ângulos internos de um polígono convexo medem
> 130 graus cada um e os demais ângulos internos medem
> 128 graus cada um . O numero de lados desse polígono é:
>
>a)6 b) 7 c)13   d )16 e)17
>
>
>  Agradeço desde de já.


=
Instruções para entrar na lista, sair da lista e usar a lista em
http://www.mat.puc-rio.br/~nicolau/olimp/obm-l.html
=


Re: [obm-l] cartoes

2004-04-05 Por tôpico Rafael
Eu entendi o que você quer dizer, mas isso contradiz o enunciado. Veja:

"Se o número deste último cartão for menor do que o do cartão
obtido na retirada imediatamente anterior, devolve-se o cartão obtido na
retirada imediatamente anterior para a urna."

Ou seja, mesmo C5 já sendo o último cartão que faltava a ser retirado, como
C4 é retirado, somos obrigados a devolver C5 e retirá-lo novamente.




- Original Message -
From: <[EMAIL PROTECTED]>
To: <[EMAIL PROTECTED]>
Sent: Monday, April 05, 2004 1:39 PM
Subject: Re: [obm-l] cartoes


quero dizer (nao sei nem se disse errado), na rodada 14 temos C1, C2, C3, C5
+ C4... Voltaria C5 e retornaria a mesa na rodada 15, mas ja nao estava
sobre ela na 14?


=
Instruções para entrar na lista, sair da lista e usar a lista em
http://www.mat.puc-rio.br/~nicolau/olimp/obm-l.html
=


Re: [obm-l] cartoes

2004-04-05 Por tôpico Rafael
Seja Cn o enésimo cartão, supondo que C5 seja retirado, qualquer outro será
menor e fará que ele seja devolvido. Por hipótese, visto que queremos o
número máximo de retiradas, retiraremos C4 e C5 volta para a urna. Depois,
retiramos C3 e C4 volta para a urna... Esquematizando:

C5 --> C4 --> C3 --> C2 --> C1

Não há um cartão menor que C1, este permanece. Novamente:

C5 --> C4 --> C3 --> C2

Não há um cartão menor que C2 que ainda possa ser retirado, este permanece.

C5 --> C4 --> C3

C5 --> C4

C5


Logo, no máximo, serão feitas 15 retiradas.




- Original Message -
From: [EMAIL PROTECTED]
To: [EMAIL PROTECTED]
Sent: Monday, April 05, 2004 1:32 AM
Subject: [obm-l] cartoes


Ola pessoal,

Como voces resolveriam esta ?


1- Uma urna contém cinco cartões,numerados com 1, 2, 3, 4, 5. Retira-se, ao
acaso, um cartão da urna colocando-osobre a mesa. A seguir, retira-se um
outro cartão. Se o número deste último cartão for menor do que o do cartão
obtido na retirada imediatamente anterior,devolve-se o cartão obtido na
retirada imediatamente anterior para a urna. A seguir, retira-se mais um
cartão, e repete-se o procedimento até que todos oscinco cartões estejam
sobre a mesa. O número máximo de retiradas de cartões que matematicamente
garante que todos os cinco cartões estarão sobre a mesa é igual a:

a) 5
b) 10
c) 15
d) 20
e) 25

=
Instruções para entrar na lista, sair da lista e usar a lista em
http://www.mat.puc-rio.br/~nicolau/olimp/obm-l.html
=


Re: [obm-l] logaritmo

2004-04-04 Por tôpico Rafael



Talvez, um pouquinho mais simples:
 
3^log(2,1/3) = 3^[log(2,3)/log(1/3,3)] = 
3^[-log(2,3)] = 3^[log(1/2,3)] = 1/2
 
 
 

  - Original Message - 
  From: 
  [EMAIL PROTECTED] 
  
  To: [EMAIL PROTECTED] 
  Sent: Sunday, April 04, 2004 5:56 
PM
  Subject: Re: [obm-l] logaritmo
  Vou *mexer* primeiro no expoente, que eh log[1/3] 2. Vamos 
  trasforma-lo para a base 10: log[1/3] 2 = log[10] 2 / log[10] (1/3) = 
  log[10] 2 / log[10] (3^(-1)) =  0,3010... / ((-1)*(0,4771)) = = 
  0,3010... / (-0,4771) = - 0,6308 ... Entao: 3^log[1/3] 2 = 
  3^(- 0,6308 ...) = (aprox) 1/2 Em uma mensagem de 4/4/2004 
  17:09:35 Hora padrão leste da Am. Sul, [EMAIL PROTECTED] escreveu: 
  
  Alguem sabe resolver essa: 3^log1/3 2 a base é 
1/3 (1 sobre 3) 



Re: [obm-l] congruencia e aritmetica modular

2004-04-04 Por tôpico Rafael



Sim, foi exatamente essa a minha interpretação. Na 
ajuda da calculadora, como se vê, é dito que se trata do resto de x/y. Mas 
se o conceito lá exposto estiver errado, então, conseqüentemente, eu errei na 
explicação.
 
 

  - Original Message - 
  From: 
  [EMAIL PROTECTED] 
  
  To: [EMAIL PROTECTED] 
  Sent: Sunday, April 04, 2004 5:44 
PM
  Subject: Re: [obm-l] congruencia e 
  aritmetica modular
  Eu entendi o que o Rafael estava querendo dizer com 9 
  mod 4 = 1, pois 9 / 4 = 2 e resto 1. Quem usa o Windows sabe que tem 
  aquela calculadora, certo ? Eh so clicar em *mod* com o lado direito do mouse 
  e aparecerah *o que eh isto*, clique com o esquerdo do mouse. Para quem 
  estiver com preguica vou postar aqui o que eh dito: 
   Tecla Mod Exibe o módulo, ou o resto, 
  de x/y. Use este botão como um operador binário. Por exemplo, para 
  calcular o módulo de 5 dividido por 3, clique em 5 MOD 3 =, que é igual a 2. 
  Equivalente de teclado = % ** 



Re: [obm-l] probleminhas...

2004-04-03 Por tôpico Rafael
Marlen,


Eu me enrolei exatamente com esse problema no mês passado. Leia:

http://www.mat.puc-rio.br/~nicolau/olimp/obm-l.200403/msg00252.html

Aí você vai encontrar a demonstração do Cláudio, incrivelmente clara!


Um forte abraço,

Rafael de A. Sampaio





- Original Message -
From: Marlen Lincoln Silva
To: [EMAIL PROTECTED]
Sent: Saturday, April 03, 2004 11:46 PM
Subject: [obm-l] probleminhas...



Por favor, uma ajudinha neste problema em que me enrolei...
Prove que quaisquer que sejam os números a e b, mdc (a,b)  x  mmc (a,b) = a
x b



=
Instruções para entrar na lista, sair da lista e usar a lista em
http://www.mat.puc-rio.br/~nicolau/olimp/obm-l.html
=


[obm-l] Re: [obm-l] Re: [obm-l] dúvidas - Correcao

2004-04-03 Por tôpico Rafael
Vou tentar explicar de uma outra forma, ainda que algumas semelhanças sejam
inevitáveis.

(2^(1/3) + 3^(1/2))^10 =
= Sum[C(10,k) * 2^[(1/3)*(10-k)] * 3^[(1/2)*k], {k, 0, 10}] =
= Sum[C(10,k) * 2^[(10-k)/3] * 3^(k/2), {k, 0, 10}]

Não é muito difícil demonstrar, por redução ao absurdo, que a raiz enésima
de um número primo é irracional. Assim, como 2 e 3 são primos, precisamos
que os seus expoentes sejam inteiros. Temos que k varia de 0 a 10, então:

k/2 é inteiro ==> k pertence a {0,2,4,6,8,10}
(10-k)/3 é inteiro ==> k = 4 ou k = 10

Seja T_n o enésimo termo do desenvolvimento de (x+y)^n,

k = 4 ==> T_5 = C(10,4) * 2^[(10-4)/3] * 3^(4/2) = 7560
k = 10 ==> T_11 = C(10,10) * 2^[(10-10)/3] * 3^(10/2) = 243

Lembre-se de que o desenvolvimento de (x+y)^n tem n+1 termos. Assim, o
quinto e o décimo primeiro termos são os únicos cujo valor é racional.


Abraços,

Rafael de A. Sampaio




- Original Message -
From: TSD
To: [EMAIL PROTECTED]
Sent: Saturday, April 03, 2004 9:07 AM
Subject: [obm-l] Re: [obm-l] dúvidas - Correcao


a 2º questão poderia explicar melhor por favor.

=
Instruções para entrar na lista, sair da lista e usar a lista em
http://www.mat.puc-rio.br/~nicolau/olimp/obm-l.html
=


Re: [obm-l] Genios

2004-04-03 Por tôpico Rafael
Title: Genios



Ainda assim, visto que errar é algo que ocorre 
com todos nós, você não deixa de ser também um dos participantes mais 
interessados pelos problemas alheios, dispondo do seu tempo e conhecimento 
para responder aos e-mails e tentar ajudar, com bom humor ainda... 
;-)
 
 

  - Original Message - 
  From: 
  Claudio Buffara 
  To: [EMAIL PROTECTED] 
  Sent: Saturday, April 03, 2004 8:59 
  PM
  Subject: [obm-l] Genios
  Oi, Fael:Se voce reparar 
  bem, vai ver que eu sou o participante da lista que mais manda mensagens com 
  duvidas ou solucoes erradas e/ou incompletas. Assim, se esse "GENIOS" 
  se refere a minha pessoa, entao soh pode ser a sigla de:Geralmente 
  Enganado Nas Interpretacoes Ou 
Solucoes.[]s,Claudio.


Re: [obm-l] Funcao composta!

2004-04-03 Por tôpico Rafael
Fábio,

Para a questão, a sua dificuldade não tem importância, mas, para você, creio
que sim. Assim, vou mostrar duas formas:

Você pode encontrar f(x), o que é mais trabalhoso:

f(g(x)) = 4x^2 - 8x + 6
g(x) = 2x - 1

f(2x-1) = 4x^2 - 8x + 6

t = 2x - 1 ==> x = (t+1)/2

f(2*(t+1)/2 - 1) = 4((t+1)/2)^2 - 8(t+1)/2 + 6
f(t) = (t+1)^2 - 4(t+1) + 6
f(x) = x^2 + 2x + 1 - 4x - 4 + 6 = x^2 - 2x + 3

f(2) = 2^2 - 2*2 + 3 = 4 - 4 + 3 = 3


Ou ainda, mais rapidamente,

f(g(x)) = 4x^2 - 8x + 6
g(x) = 2x - 1

g(x) = 2x - 1 = 2 ==> x = 3/2

f(g(3/2)) = f(2) = 4(3/2)^2 - 8(3/2) + 6 = 9 - 12 + 6 = 3


Abraços,

Rafael de A. Sampaio



- Original Message -
From: Fabio Contreiras
To: [EMAIL PROTECTED]
Sent: Saturday, April 03, 2004 7:12 PM
Subject: [obm-l] Funcao composta!


ola pessoal...
nao to conseguindo chegar em f(x), so consigo quando rola que tenho que
igualar f(x) = ax + b

esse aki tem funcao do 2o grau...
como chego nela?

abracos!


2 )  Se f ( g ( x ) ) = 4 x2 - 8x + 6 e g ( x ) = 2x - 1, então f ( 2 ) é
igual a:

=
Instruções para entrar na lista, sair da lista e usar a lista em
http://www.mat.puc-rio.br/~nicolau/olimp/obm-l.html
=


Re: [obm-l] congruencia e aritmetica modular

2004-04-03 Por tôpico Rafael
Creio que, para ambas as perguntas, a resposta seja sim.


De acordo com a definição:

A = B (mod n) <==> (A-B)/n é inteiro


- Exemplo:

6 = 2 (mod 4), pois (6-2)/4 = 1 que é inteiro


Para a segunda pergunta:

Seja B = q*n + r  e  0 =< r < n,

B mod n = r


- Exemplo:

9 mod 4 = 1, pois 9 / 4 = 2 e resto 1.


Abraços,

Rafael de A. Sampaio



- Original Message -
From: "André Zimmermann" <[EMAIL PROTECTED]>
To: <[EMAIL PROTECTED]>
Sent: Saturday, April 03, 2004 2:52 PM
Subject: [obm-l] congruencia e aritmetica modular



Pessoal,

É satisfatório e suficiente dizer que:

A é congruente a B (módulo n) se n for divisor da diferença entre A e B ?

E que B módulo n é igual ao resto da divisão inteira de B por n ?


Estas são as dúvidas de um cérebro enferrujado

Obrigado pelo desengripante.


André.

=
Instruções para entrar na lista, sair da lista e usar a lista em
http://www.mat.puc-rio.br/~nicolau/olimp/obm-l.html
=


Re: [obm-l] Problema de Estatística

2004-04-02 Por tôpico Rafael
Falta só um não:
Em uma pesquisa com duas perguntas A e B 
30 pessoas responderam  sim para as duas questões  
60 não para A
80 NÃO para B
130 responderam sim para pelo menos uma questão
Pergunta-se o número de entrevistados foi de ?
Obs nenhuma pergunta ficou sem resposta;

Aí dá pra resolver e a resposta é 150. Dica, chame de
x o número de pessoas que respondeu sim para B e não
para A e faça uma tabela com os dados assim:

   | BS | BN  |
---||-|
AS | 30 |100-x|
---||-|
AN |  x |60-x | 60
---||-|
   || 80  |

Boa sorte!

Abraços,

Rafael.

 --- persio ca <[EMAIL PROTECTED]> escreveu: >
Pessoal este problema está correto,ou falta
> informações
>  
> Em uma pesquisa com duas perguntas A e B 
> 30 pessoas responderam  sim para as duas questões  
> 60 não para A
> 80 para  B
> 130 responderam sim para pelo menos uma questão
> Pergunta-se o número de entrevistados foi de ?
> Obs nenhuma pergunta ficou sem resposta;
> 
> Agradeço desde já

__

Yahoo! Mail - O melhor e-mail do Brasil! Abra sua conta agora:
http://br.yahoo.com/info/mail.html
=
Instruções para entrar na lista, sair da lista e usar a lista em
http://www.mat.puc-rio.br/~nicolau/olimp/obm-l.html
=


[obm-l] Pentágono, hexágono e um ângulo: Geometria

2004-04-02 Por tôpico Rafael
Pessoal,

Durante um "passeio" pela internet, entre outros problemas de Geometria,
fiquei curioso por este:


Um pentágono regular é interno a um hexágono regular. Os lados do hexágono e
do pentágono são iguais, sendo que o lado AB é lado do pentágono e do
hexágono. Seja D o vértice do pentágono oposto ao lado AB e F o vértice do
hexágono oposto ao vértice A, qual é o valor do ângulo CDF, em que C é o
vértice consecutivo de B?


Estive pensando sobre ele, mas ainda não "enxerguei" um caminho, ou alguma
construção auxiliar que ajudasse. Se alguém tiver alguma sugestão ou mesmo
conhecer o problema, ficarei grato por qualquer comentário, pois sequer
havia a resposta dele no site.


Obrigado,

Rafael de A. Sampaio

=
Instruções para entrar na lista, sair da lista e usar a lista em
http://www.mat.puc-rio.br/~nicolau/olimp/obm-l.html
=


Re: [obm-l] Somatorios de k^6 e de k^8

2004-03-30 Por tôpico Rafael
Na verdade, quando escrevi: "O mesmo seria válido para: 0/0, 0*oo, oo/oo,
1^oo, oo - oo.", referia-me à indeterminação dessas expressões, assim como
acontece com 0^0.



- Original Message -----
From: "Rafael" <[EMAIL PROTECTED]>
To: "OBM-L" <[EMAIL PROTECTED]>
Sent: Wednesday, March 31, 2004 12:10 AM
Subject: Re: [obm-l] Somatorios de k^6 e de k^8


É verdade, Nicolau, para o proposto, não houve qualquer erro. Entretanto,
lendo com mais atenção, surgiram-me duas perguntas:

1) Qual é a "vantagem" de se calcular a soma até n (exclusive)?

2) Sobre a definição proposta: S_m(n) = 0^m + 1^m + ... + (n-1)^m, em sua
mensagem anterior, é considerado que S_0(n) = n. Isso só é verdade, de
acordo com a definição, se 0^0 = 1, o que é uma convenção. Lembro-me de já
ter lido que nem sempre é possível afirmar isso, ou melhor, que somente uma
função analítica permite a conclusão, em geral, de que 0^0 = 1. O mesmo
seria válido para: 0/0, 0*oo, oo/oo, 1^oo, oo - oo. Você poderia explicar e
dar detalhes sobre isso?


Muito obrigado,

Rafael de A. Sampaio


=
Instruções para entrar na lista, sair da lista e usar a lista em
http://www.mat.puc-rio.br/~nicolau/olimp/obm-l.html
=


Re: [obm-l] Somatorios de k^6 e de k^8

2004-03-30 Por tôpico Rafael
É verdade, Nicolau, para o proposto, não houve qualquer erro. Entretanto,
lendo com mais atenção, surgiram-me duas perguntas:

1) Qual é a "vantagem" de se calcular a soma até n (exclusive)?

2) Sobre a definição proposta: S_m(n) = 0^m + 1^m + ... + (n-1)^m, em sua
mensagem anterior, é considerado que S_0(n) = n. Isso só é verdade, de
acordo com a definição, se 0^0 = 1, o que é uma convenção. Lembro-me de já
ter lido que nem sempre é possível afirmar isso, ou melhor, que somente uma
função analítica permite a conclusão, em geral, de que 0^0 = 1. O mesmo
seria válido para: 0/0, 0*oo, oo/oo, 1^oo, oo - oo. Você poderia explicar e
dar detalhes sobre isso?


Muito obrigado,

Rafael de A. Sampaio




- Original Message -
From: "Nicolau C. Saldanha" <[EMAIL PROTECTED]>
To: <[EMAIL PROTECTED]>
Sent: Tuesday, March 30, 2004 4:42 PM
Subject: Re: [obm-l] Somatorios de k^6 e de k^8


On Tue, Mar 30, 2004 at 03:58:06PM -0300, Rafael wrote:
> A fórmula que você citou não seria a fórmula de Faulhaber?
>
> http://mathworld.wolfram.com/FaulhabersFormula.html

Eu não conhecia este nome, mas acabo de olhar a página que você
indicou e sim, é basicamente a mesma fórmula.

> Além disso, os sinais dos termos na "segunda coluna", ou muito me engano,
ou
> são *positivos*:

Você deve estar somando até n inclusive.
O livro que eu citei, na seção que eu citei, soma até n *exclusive*,
conforme eu defini na mensagem:

>> Escreva S_m(n) = 0^m + 1^m + ... + (n-1)^m.

[]s, N.


=
Instruções para entrar na lista, sair da lista e usar a lista em
http://www.mat.puc-rio.br/~nicolau/olimp/obm-l.html
=


Re: [obm-l] EsSA 2003

2004-03-30 Por tôpico Rafael
Elton,

Embora simples, essa é uma questão interessante, pois tem por objetivo
enfocar a proximidade dos sistemas lineares e da Geometria Analítica.

A alternativa (a) está incorreta. Por quê? Se as duas equações são uma mesma
reta, então não temos duas equações, e sim uma! Para uma equação a duas
incógnitas, o sistema é possível e indeterminado.

A alternativa (b) está errada, pois retas paralelas têm intersecção no
infinito. Logo, o sistema formado por elas é impossível.

A alternativa (c) é falsa, porque retas concorrentes têm um único ponto em
comum. Trata-se de um sistema possível e determinado.

A alternativa (d) é verdadeira e a justificativa é exatamente a da
alternativa (a).

A alternativa (e) também está incorreta e a justificativa é a mesma da
alternativa (c).


Abraços,

Rafael de A. Sampaio




- Original Message -
From: "elton francisco ferreira" <[EMAIL PROTECTED]>
To: <[EMAIL PROTECTED]>
Sent: Tuesday, March 30, 2004 7:19 PM
Subject: [obm-l] EsSA 2003


Considerando um sistema de duas equações com duas
incógnitas, assinale a alternativa correta:

a) Se as equações são representadas por uma mesma reta,
então o sistema é determinado.

b) Se as equações são representadas por retas
paralelas, então o sistema é indeterminado.

c) Se as equações são representadas retas concorrentes,
então o sistema é indeterminado.

d) Se as equações são representadas retas
coincidentes, então o sistema é indeterminado.

e) Se as equações são representadas retas concorrentes,
então o sistema é impossível.


=
Instruções para entrar na lista, sair da lista e usar a lista em
http://www.mat.puc-rio.br/~nicolau/olimp/obm-l.html
=


Re: RES: RES: [obm-l] CR(n,p) = C(n+p-1,p)

2004-03-30 Por tôpico Rafael
Na UFPE vende. Vá na área 2, na secretaria da
matemática, onde se faz matrícula vende. Pelo menos
vendia em 2002...

Abraços,

Rafael.

 --- Claudio Buffara <[EMAIL PROTECTED]>
escreveu: > on 29.03.04 23:17, David M. Cardoso at
> [EMAIL PROTECTED] wrote:
> 
> > 
> > Bem.. eu moro em Recife/PE.. se não tiver como eu
> comprar por aqui,
> > eu vo ter q esperar meu pai ir pra sp, assim ele
> traz pra mim, mas se esse
> > for o caso eu vou precisar saber dizer pra ele
> onde ele tem que ir pra
> > comprar esse livro pra mim..
> > 
> > Agradeço muito a boa vontade..
> > David
> > 
> > 
> Nao precisa esperar o seu pai vir pra SP.
> 
> Entre no site:
> http://www.sbm.org.br/
> 
> No menu do lado esquerdo clique em "Livros".
> 
> Em seguida, clique em "Colecao do Professor de
> Matematica (CPM)"
> 
> O livro que voce quer eh o CPM/02. Clique para ver
> uma descricao.
> 
> Para encomendar, volte a segunda pagina, clique em
> "Como e onde comprar" e
> siga as instrucoes lah contidas.
> 
> 
> []s,
> Claudio.
> 
> 
>
=
> Instruções para entrar na lista, sair da lista e
> usar a lista em
> http://www.mat.puc-rio.br/~nicolau/olimp/obm-l.html
>
= 

__

Yahoo! Mail - O melhor e-mail do Brasil! Abra sua conta agora:
http://br.yahoo.com/info/mail.html
=
Instruções para entrar na lista, sair da lista e usar a lista em
http://www.mat.puc-rio.br/~nicolau/olimp/obm-l.html
=


Re: [obm-l] Somatorios de k^6 e de k^8

2004-03-30 Por tôpico Rafael
Nicolau,

A fórmula que você citou não seria a fórmula de Faulhaber?

http://mathworld.wolfram.com/FaulhabersFormula.html

Além disso, os sinais dos termos na "segunda coluna", ou muito me engano, ou
são *positivos*:

S_0(n) = n
S_1(n) = 1/2 n^2 + 1/2 n
S_2(n) = 1/3 n^3 + 1/2 n^2 + 1/6 n
S_3(n) = 1/4 n^4 + 1/2 n^3 + 1/4 n^2 + 0 n
S_4(n) = 1/5 n^5 + 1/2 n^4 + 1/3 n^3 + 0 n^2 - 1/30 n
S_5(n) = 1/6 n^6 + 1/2 n^5 + 5/12 n^4 + 0 n^3 - 1/12 n^2 + 0 n
...


Abraços,

Rafael de A. Sampaio




- Original Message -
From: "Nicolau C. Saldanha" <[EMAIL PROTECTED]>
To: <[EMAIL PROTECTED]>
Sent: Tuesday, March 30, 2004 9:00 AM
Subject: Re: [obm-l] Somatorios de k^6 e de k^8



Outros já responderam a pergunta do Morgado, então eu vou responder
a pergunta que o Morgado *não* fez, que é como calcular estas coisas *sem*
usar o Maple. Estou quase copiando a seção 6.5 do Matemática Concreta,
de Graham, Knuth e Patashnik que fala de números de Bernoulli.

Escreva S_m(n) = 0^m + 1^m + ... + (n-1)^m.
Não é difícil provar que

S_m(n) = (1/(m+1)) SOMA_{0 <= k <= m} binomial(m+1,k) B_k n^{m+1-k}

onde B_k são os números de Bernoulli, com valores iguais a
1, -1/2, 1/6, 0, -1/30, 0, 1/42, 0, -1/30, 0, 5/66, 0, -691/2730, ...
e que podem ser definidos por

z/(e^z - 1) = SOMA_{n >= 0} B_n z^n/n!.

Usando a fórmula para S_m, não é difícil montar uma tabela de coeficientes.

S_0(n) = n
S_1(n) = 1/2 n^2 - 1/2 n
S_2(n) = 1/3 n^3 - 1/2 n^2 + 1/6 n
S_3(n) = 1/4 n^4 - 1/2 n^3 + 1/4 n^2 + 0 n
S_4(n) = 1/5 n^5 - 1/2 n^4 + 1/3 n^3 + 0 n^2 - 1/30 n
S_5(n) = 1/6 n^6 - 1/2 n^5 + 5/12 n^4 + 0 n^3 - 1/12 n^2 + 0 n

Em cada coluna, os coeficientes são dados por uma fórmula bem simples:
uma constante misteriosa (o número de Bernoulli) vezes um binomial.
Por exemplo, o coeficiente de n^(m+1) é 1/(m+1), o de n^m é -1/2,
o de n^(m-1) é (1/(m+1)) binomial(m+1,2) B_2 = m/12, o de n^(m-2) é 0,
o de n^(m-3) é (1/(m+1)) binomial(m+1,4) B_4 = -m(m-1)(m-2)/720
e o de n^(m-4) é 0. Ajuda muito saber que B_n = 0 para n ímpar e maior que
1.
Mas o fato é que esta tabela triangular nos dá a cada passo o novo B_n,
basta usar o fato óbvio que S_m(1) = 0 para m > 0.

[]s, N.


=
Instruções para entrar na lista, sair da lista e usar a lista em
http://www.mat.puc-rio.br/~nicolau/olimp/obm-l.html
=


[obm-l] Re: [obm-l] Simplificação

2004-03-28 Por tôpico Rafael



Se o exercício é mesmo assim...
 
 
2^31 = 2 147 483 648
3^31 = 617 673 396 283 947
-
2^31 + 3^31 = 617 675 543 767 595
 
 
2^29 = 536 870 912
3^29 = 68 630 377 364 883

2^29 + 3^29 = 68 630 914 235 795
 
 
mdc(617 675 543 767 595 ; 68 630 914 235 795) 
= 5
 
 
Assim: 123535108753519 / 13726182847159 é fração 
irredutível.
 
 
 

  - Original Message - 
  From: 
  Fábio Bernardo 
  
  To: OBM 
  Cc: [EMAIL PROTECTED] ; [EMAIL PROTECTED] 
  
  Sent: Sunday, March 28, 2004 1:03 
PM
  Subject: [obm-l] Simplificação
  
  Pessoal, ajudem-me nesse por favor.
   
  Simplifique a fração:
   
  (2^31+3^31)/(2^29+3^29)
   
  Desde já agradeço.
   
   


Re: [obm-l] Rafael

2004-03-28 Por tôpico Rafael



Eu não disse em qualquer momento que elas são 
inapropriadas. Somente repetir as mensagens não me parece o mais 
adequado...
 
Já responderam à sua questão sobre porcentagem, 
creio que você já tenha tido a sua dúvida respondida.
 
 
Abraços,
 
Rafael de A. Sampaio
 
 
 

  - Original Message - 
  From: 
  TSD 
  To: [EMAIL PROTECTED] 
  Sent: Sunday, March 28, 2004 3:20 
PM
  Subject: [obm-l] Rafael
  
  pow. banido por quê ? eu só quero tirar a minha 
  dúvida´... é pedir de+ ?
  eu sei que ninguém ganha para isso, mas eu 
  agradeço a todos pela boa vontade. e a respeito de minhas questões elas são 
  apropriadas sim , pois as mesmas são de matemática! ao contrário de gente 
  que vem colocando questões de física no 
canal.!!!


[obm-l] Re: [obm-l] Re: [obm-l] análise combinatória

2004-03-27 Por tôpico Rafael
Ahhh, agora faz sentido lar do Coelhinho da Páscoa, claro...!

Ainda assim, duas pequenas correções sobre o que você escreveu:

- "Chará" escreve-se com 'x', portanto, você, provavelmente, é meu *xará*;
- "Passárgada" escreve-se com apenas um 's', veja: Pasárgada.


Sim, não é só de Matemática que gosto na vida, felizmente... ;-)


Abraços,

Rafael de A. Sampaio




- Original Message -
From: "seanjr" <[EMAIL PROTECTED]>
To: <[EMAIL PROTECTED]>
Sent: Saturday, March 27, 2004 10:50 PM
Subject: [obm-l] Re: [obm-l] análise combinatória


Obrigado.

Vc é meu chará e R$ é a moeda imaginária, rafaéis, de uma
nação insular na costa de Passárgada. Lar do Coelhinho da
páscoa. =P




=
Instruções para entrar na lista, sair da lista e usar a lista em
http://www.mat.puc-rio.br/~nicolau/olimp/obm-l.html
=


[obm-l] Re: [obm-l] Re: [obm-l] análise combinatória

2004-03-27 Por tôpico Rafael
Contar o número de soluções da equação x + y + z + t = 20, tais sendo
inteiras e *não-negativas*, como muito bem me lembrou o Prof. Morgado,
equivale ao número de combinações completas de 4 elementos escolhidos 20 a
20, sendo que tais elementos (pessoas) podem aparecer repetidamente: uma
mesma pessoa pode receber mais de uma nota, ou mesmo, nenhuma.

Representando as combinações completas (ou, como preferem outros,
combinações com repetição) por *C(n,k), temos que:

*C(n,k) = C(n+k-1,k) = (n+k-1)!/(k!(n-1)!)

Assim: *C(4,20) = 23!/(20!3!) = 1771.


Abraços,

Rafael de A. Sampaio




- Original Message -
From: "Douglas Drumond" <[EMAIL PROTECTED]>
To: <[EMAIL PROTECTED]>
Sent: Saturday, March 27, 2004 8:53 PM
Subject: Re: [obm-l] Re: [obm-l] análise combinatória


Rafael escreveu:
 > Sejam x, y, z, t as quatro pessoas em questão, teremos
 > x + y + z + t = 20
 >Para contar o número de soluções dessa equação, tais sendo inteiras e
 > positivas, faz-se: 23!/(3!20!) = 1771 maneiras diferentes

Por que? Nao consegui entender o porque de 23!/(3!20!)

=
Instruções para entrar na lista, sair da lista e usar a lista em
http://www.mat.puc-rio.br/~nicolau/olimp/obm-l.html
=


[obm-l] Re: [obm-l] análise combinatória

2004-03-27 Por tôpico Rafael
Sejam x, y, z, t as quatro pessoas em questão, teremos:

x + y + z + t = 20


Para contar o número de soluções dessa equação, tais sendo inteiras e
positivas, faz-se:

23!/(3!20!) = 1771 maneiras diferentes

..


Curiosidade: algum país deste mundo (ou de outro) usa notas de R$
333,33. Dá inveja de tanta criatividade...


Abraços,

Rafael de A. Sampaio





- Original Message -
From: "seanjr" <[EMAIL PROTECTED]>
To: <[EMAIL PROTECTED]>
Sent: Saturday, March 27, 2004 5:59 PM
Subject: [obm-l] análise combinatória


De qts maneiras diferentes é possível distribuir 20 notas de
R$333,33 para 4 pessoas?


=
Instruções para entrar na lista, sair da lista e usar a lista em
http://www.mat.puc-rio.br/~nicolau/olimp/obm-l.html
=


[obm-l] Re: [obm-l] grafico de funçoes

2004-03-27 Por tôpico Rafael



Dê uma olhada em http://www.peda.com/
 
Se você só pretende fazer gráficos, o GrafEq deve 
ser suficiente...
 
 
 
Abraços,
 
Rafael de A. Sampaio
 
 
 

  - Original Message - 
  From: 
  [EMAIL PROTECTED] 
  To: [EMAIL PROTECTED] 
  Sent: Saturday, March 27, 2004 6:35 
  PM
  Subject: [obm-l] grafico de funçoes
  sabem dizer se 
  tem algum programa de computador (download gratis)que dado uma função o 
  programa faça o grafico. por favor nome ou 
  site.obrigado. 


Re: [obm-l] 2^n ? pq ?

2004-03-27 Por tôpico Rafael
Se o assunto é Matemática, não vejo por que não ser pertinente à lista.
Entretanto, só não escreva e-mails com assunto "dúvidas". Creio que se possa
definir melhor sobre o que se está escrevendo, isso é ótimo para quem
acompanha as mensagens, acredite.

Sobre o que você quer provar, a demonstração mais freqüente é a que se faz
por Análise Combinatória, mas mostrarei as duas formas.

Pelo Princípio da Indução Finita (PIF):

Se n = 1, então temos um conjunto A que possui 1 elemento, logo o conjunto
das partes (ou subconjuntos) de A tem dois elementos: o subconjunto vazio e
o subconjunto com o único elemento de A. Logo, n[P(A)] = 2^1 = 2 é
verdadeiro.

Agora, supondo-se que a propriedade seja verdadeira para n = p, provar-se-á
que ela é verdadeira para n = p + 1. Assim:

Hipótese: n = p ==> n[P(A)] = 2^p
Tese: n = p + 1 ==> n[P(A)] = 2^(p+1)

n = p ==> n[P(A)] = 2^p

Somando-se 1 na primeira igualdade, dobra-se n[P(A)], pois somar 1 ao valor
de n significa adicionar um elemento ao conjunto A; aos subconjuntos
anteriormente formados poder-se-á acrescentar ou não o elemento que
adicionamos ao conjunto A, o que nos dá o dobro de possibilidades. Desse
modo:

n = p + 1 ==> n[P(A)] = 2 * 2^p
n = p + 1 ==> n[P(A)] = 2^(p+1)

(c.q.d.)


A outra forma, por Análisa Combinatória, justifica a fórmula. Suponha:

A = {1, 2, 3, 4, 5, ..., n}

Quais seriam os subconjuntos possíveis de A? Primeiramente, o número de
combinações dos elementos de A escolhidos um a um; posteriormente, as
combinações dos elementos de A escolhidos dois a dois; e, analogamente, até
todos os elementos serem escolhidos. Dois detalhes são importantes: a ordem
de escolha desses elementos não importa para a formação do subconjunto, por
isso usamos as combinações; e, por fim, lembre-se de que o conjunto vazio é
subconjunto de qualquer conjunto. Matematicamente:

C(n,0) + C(n,1) + C(n,2) + C(n,3) + ... + C(n,n)

em que C(n,k) = n!/[k!(n-k)!] são as combinações de n elementos
tomados k a k.

C(n,0)  dos n elementos não se escolhe qualquer um (*)
C(n,1)  dos n elementos escolhe-se um deles
..
C(n,n-1)  dos n elementos escolhem-se n-1 elementos
C(n,n)  dos n elementos escolhem-se todos eles (**)


* Subconjunto vazio.

** Todo o conjunto está contido nele próprio, por isso é subconjunto de si
mesmo.


Pelo teorema do binômio de Newton, temos:

(x + y)^n = Sum[C(n,k) * x^(n-k) * y^k, {k, 0, n}]

que significa que se estão somando as parcelas C(n,k)*x^(n-k)*y^k,
com k variando de 0 a n.

Fazendo x = y = 1, teremos:

(1 + 1)^n = Sum[C(n,k) * 1^(n-k) * 1^k, {k, 0, n}]

2^n = Sum[C(n,k), {k, 0, n}] = C(n,0) + C(n,1) + C(n,2) + ... + C(n,n)

(c.q.d.)


Espero ter podido esclarecer a sua dúvida e seja bem-vindo à lista.


Abraços,

Rafael de A. Sampaio





- Original Message -
From: <[EMAIL PROTECTED]>
To: <[EMAIL PROTECTED]>
Sent: Saturday, March 27, 2004 5:33 PM
Subject: [obm-l] 2^n ? pq ?


oi pessoal, sou novo na lista e nao sei se o assunto eh pertinente:

ten um exercicio no livro 1 da colecao fundamentos de matematica
elementar, q pede o seguinte:

seja um conjunto A com n elementos. O conjunto P(A) tem 2^n elementos.
Prove pelo principio da inducao finita.

alguem poderia me ajudar ?



=
Instruções para entrar na lista, sair da lista e usar a lista em
http://www.mat.puc-rio.br/~nicolau/olimp/obm-l.html
=


[obm-l] Re: [obm-l] Re: RES: [obm-l] dúvida

2004-03-27 Por tôpico Rafael



Talvez, fique mais claro dizer que a arma faz 30° e 
60° com a horizontal, em vez de 30° e 60° com o solo. Assim, evita-se uma 
interpretação dúbia sobre a altura do observador estar sendo 
desprezada.
 
 

  - Original Message - 
  From: 
  Rafael 
  
  To: [EMAIL PROTECTED] 
  Sent: Saturday, March 27, 2004 5:03 
  PM
  Subject: [obm-l] Re: RES: [obm-l] 
  dúvida
  
  Na verdade, a altura 'h' é a distância do solo 
  até a reta que contém a trajetória do pato, suposto em movimento retilíneo e 
  uniforme. Por isso, tanto para uma inclinação quanto para a outra da arma, a 
  distância 'h' é a mesma, visto que só se pretende calcular os segmentos de 
  reta percorridos até o instante em que a arma faz 60° com o solo e, 
  posteriormente, até o instante que a arma faz 30° com o solo. A diferença 
  das medidas desses segmentos é, precisamente, o comprimento da 
  distância que o pato percorreu do instante em que foi "quase" 
  atingido até ser atingido pelo segundo projétil.
   
   
  Abraços,
   
  Rafael de A. Sampaio
   


[obm-l] Re: RES: [obm-l] dúvida

2004-03-27 Por tôpico Rafael
Se estivéssemos calculando a distância do pássaro até o solo, precisaríamos
desprezar a altura do observador. Tendo-se a distância do observador ao
pássaro e o ângulo (com a horizontal) que o observador enxerga o pássaro,
aplicaríamos a função seno e o problema estaria resolvido. Reitero: a altura
do observador é desprezada em comparação à distância do pássaro ao solo.

Para o problema em questão, isso é irrelevante, pois a distância calculada é
a percorrida pelo pato "no ar". Não é comparada ou desprezada a altura do
observador (ou o ângulo que este enxerga) com a distância do pássaro ao
solo.

Embora esses problemas sejam clássicos da trigonometria, muitas vezes o
excesso de cuidado para alguns exercícios é desnecessário para outros.


Abraços,

Rafael de A. Sampaio





- Original Message -
From: <[EMAIL PROTECTED]>
To: <[EMAIL PROTECTED]>
Sent: Saturday, March 27, 2004 4:50 PM
Subject: Re: RES: [obm-l] dúvida


Me chama atenção que não está sendo considerado o fato do homem não estar
com a arma nos pés...


=
Instruções para entrar na lista, sair da lista e usar a lista em
http://www.mat.puc-rio.br/~nicolau/olimp/obm-l.html
=


[obm-l] Re: RES: [obm-l] dúvida

2004-03-27 Por tôpico Rafael



Na verdade, a altura 'h' é a distância do solo até 
a reta que contém a trajetória do pato, suposto em movimento retilíneo e 
uniforme. Por isso, tanto para uma inclinação quanto para a outra da arma, a 
distância 'h' é a mesma, visto que só se pretende calcular os segmentos de reta 
percorridos até o instante em que a arma faz 60° com o solo e, 
posteriormente, até o instante que a arma faz 30° com o solo. A diferença 
das medidas desses segmentos é, precisamente, o comprimento da 
distância que o pato percorreu do instante em que foi "quase" 
atingido até ser atingido pelo segundo projétil.
 
 
Abraços,
 
Rafael de A. Sampaio
 
 

  - Original Message - 
  From: 
  [EMAIL PROTECTED] 
  
  To: [EMAIL PROTECTED] 
  Sent: Saturday, March 27, 2004 4:27 
  PM
  Subject: Re: RES: [obm-l] dúvida
  Esclarecam-me uma duvida: Suponho que h seja a 
  altura de um eixo imaginario (perpendicular ao solo) que vai do solo ateh a 
  bala, passando pelo corpo da ave, certo ? Logo h eh crescente no intervalo 
  [0,90º[, ou seja, a inclinacao da arma produz um angulo alfa, cujo cateto 
  oposto eh h, certo ? Entao quanto maior for alfa (de 0 a 90, eh claro) maior 
  sera h. Mas vejamos o sistema: Tg[Pi/3] = h / x Tg[Pi/6] = h / 
  (x+d) Porque o h da primeira equacao eh o mesmo da segunda equacao ? 
  As alturas nao serao diferentes de acordo com a declividade da arma ? 
  


<    1   2   3   4   5   6   7   8   >